[ /tv/ /rf/ /vg/ /a/ /b/ /u/ /bo/ /fur/ /to/ /dt/ /cp/ /oe/ /bg/ /ve/ /r/ /mad/ /d/ /mu/ /cr/ /di/ /sw/ /hr/ /wh/ /lor/ /s/ /hau/ /slow/ /gf/ /vn/ /w/ /ma/ /azu/ /wn/ ] [ Main | Settings | Bookmarks | Music Player ]

No.90593 Reply
File: Lewis_Carroll.jpg
Jpg, 28.11 KB, 254×328 - Click the image to expand
edit Find source with google Find source with iqdb
Lewis_Carroll.jpg
Наша кафедра упорно продолжает свою работу. Здесь мы обсуждаем математику, математиков и математические университеты, а также отвечаем на вопросы доброанонов.

Приходящие за советами аноны! Подробно пишите, что именно вы желаете изучать, с какой целью, что уже известно, что не получается.

0.(9) = 1. Тут нет ошибки.

Предыдущий: >>87526
>> No.90612 Reply
>>90596
>>90579
Сумма членов бесконечной геометрической прогрессии.
>> No.90619 Reply
File: DtMwdMBRIYU.jpg
Jpg, 97.50 KB, 453×604 - Click the image to expand
edit Find source with google Find source with iqdb
DtMwdMBRIYU.jpg
>>90593
Матанодоброанон, расскажи, как ты полюбил математику. Как она тебе давалась? Как ты представляешь себе числа? Хочу увидеть от тебя откровение, чтобы погрузиться вместе с тобой в этот прекрасный мир.
>> No.90625 Reply
>>90619
> Как она тебе давалась?
Тяжело и со скрипом же. Читал, читал, читал. А потом понял, что мне нравится и что я кое-чему научился.
> Как ты представляешь себе числа?
Никак, они абстрактны. Их можно только подумать или прочувствовать.
>> No.90632 Reply
>>90612
> Сумма членов бесконечной геометрической прогрессии
lim (n->~)sigma(9/10^n) = 1.
>> No.90647 Reply
File: 59pg1010z5an.jpg
Jpg, 37.16 KB, 535×600 - Click the image to expand
edit Find source with google Find source with iqdb
59pg1010z5an.jpg
>>90646
Либо возражай с обоснованием и ссылками на литературу, либо не мешай упарываться.
>> No.90648 Reply
>>90647
Упарывайтесь, упарывайтесь.
>> No.90649 Reply
File: 59pg1010z5an.jpg
Jpg, 37.16 KB, 535×600 - Click the image to expand
edit Find source with google Find source with iqdb
59pg1010z5an.jpg
>>90648
Так и знал.
>> No.90651 Reply
>>90649
Что знал?
>> No.90660 Reply
Какое значение в математике имеет лямбда исчисление за пределами вычислительных применений? "Чистым" математикам оно вообще неинтересно?
>> No.90689 Reply
>>90660
Чистые математики - это такие звери, которые в дикой природе не встречаются. Вообще - интересно, но в теоретическом смысле.
>> No.90693 Reply
>>90619
Я математик в четвертом поколении. У меня, собсна, и вариантов других не было кроме как полюбить математику и стать математиком. Закончил математическую школу. С абсолютно твердым пониманием что я делаю и с одобрением родителей пошел на примат в вуз. Закончил его с красным дипломом. Именно тогда я понял - что такое настоящая математика. Думал пойти в аспирантуру, но решил, что сейчас мне это не нужно. Сейчас работаю математиком в коммерческой структуре. Неплохие деньги и интересная (по-настоящему) работа. Сейчас подумываю о том, чтобы пойти-таки в аспирантуру.
Как-то так.
>> No.90694 Reply
>>90619
Алсо, согласен с попсовым высказыванием о том, что настоящая математика начинается тогда, когда заканчиваются цифры.
Для меня математика - это структуры, связи, и методы их анализа.
>> No.90702 Reply
Привет математики-куны. И вот у меня скоро будет пересдача анализа, а лекции я не писал :с Вроде бы все более менее выучил, а один вопрос нет. Длина площадь объем в смысле жордана и лебега. Дают это у нас в 1м семестре. Как отвечать, если достанется? Нужно бы много накатать чего нибудь а кроме простых определений мер Ж и Л я не знаю ничего. Расскажите понятным языком про эту тему. Спасибо заранее :3
>> No.90759 Reply
>>90593
Требуется повторить школьный курс математики (поступаю на заочку, с математикой вполне дружил в школе и когда получал очное, но с тех пор, наверное, многое забыл). Одно условие — уложиться в две недели.
Посоветуйте, пожалуйста, литературу.
>> No.90771 Reply
File: 1349139322789.jpg
Jpg, 181.74 KB, 717×1000 - Click the image to expand
edit Find source with google Find source with iqdb
1349139322789.jpg
Можете пожалуйста логически упростить это. И если из вас кто-нибудь знает хорошую(-ий) статью/книгу/гайд, то скиньте её тоже.

((ab)+!c)(!a+!c)

А вот насчёт этого, не совсем ясно, так как слэши вроде бы не используются в подобных примерах.
((a*b)\c)+(a+!c)

Если это неправильно, то тогда решите пожалуйста:
((a*b)+c)+(a+!c)
((ab)c)+(a+!c)

Напишите упрощение полностью, таблицы истинностей не нужны.
>> No.90772 Reply
>>90771

Фикс. В самом первом случае

(a * b) + !c) * (!a + !c)

И в самом последнем.

((a * b) * c ) + (a + !c)
>> No.90777 Reply
>>90772
Няша, мы не репетиторы. И нам не интересно делать это - потому что элементарщина.
Если ты столкнулся с какой-то интересно проблемой и считаешь, что решить ее можно математически - вилькоммен. А делать за тебя ДЗ никому не интересно.
>> No.90786 Reply
File: xyfbz9hcAwY.jpg
Jpg, 154.30 KB, 807×646
edit Find source with google Find source with iqdb
xyfbz9hcAwY.jpg
File: steampunk-стимпан...
Jpeg, 290.35 KB, 811×891
edit Find source with google Find source with iqdb
steampunk-стимпанк-девушка-art-115088.jpeg
File: preview6e19809c95...
Jpg, 183.67 KB, 716×1000
edit Find source with google Find source with iqdb
preview6e19809c95cdbf4897a93c0e9ab6f7eb.jpg
File: KM6PDsgw_GU.jpg
Jpg, 206.64 KB, 1280×960
edit Find source with google Find source with iqdb
KM6PDsgw_GU.jpg

>>90593
Доброанон, научи ловить кайф от математики. С меня няши.
>> No.90787 Reply
>>90777
Твои бы слова да треду в заголовок.
>> No.90797 Reply
Котаны, помогите вспомнить эти чёртовы свойства логарифмов
Элементарищна, же, но никак не могу доказать, что в log[y,x]+log[x,y]=2
x будет равен y
>> No.90811 Reply
>> No.90832 Reply
File: lalkaryady.png
Png, 27.57 KB, 992×663 - Click the image to expand
edit Find source with google Find source with iqdb
lalkaryady.png
Добрач, помоги правильно решить номер. Я вроде все расписал, но пропустил момент самого ряда an, он у нас расх, вроде, верно? Тогда как это док-ть?
>> No.90836 Reply
>>90593
А, вот, просветите моё невежество.
У меня с математикой всегда было туговато. Но с поры её преподавания у меня сложилось впечатление, что суть математики в следующем:
1. Выучить основные закономерности, правила вывода там всякие, формулы, ну, и теоремы, чтоб каждый раз их не выводить заново.
2. Смотришь на задачу, и просто подбираешь цепочку преобразований, правил, теорем, которые в итоге выведут тебя к свету.
3. Профит.

Т.е. ключевой тут 2 пункт, который, по сути - задача перебора, и талант математика в том, чтобы интуитивно видеть наиболее подходящие элементы перебора.

Ответьте поскорее, я мучаюсь уже много лет по этому поводу.
>> No.90837 Reply
>>90836
Если бы задача математика заключалась в этом, вся математика давно была бы уничтожена декларативными языками программирования типа пролога (ruwiki://Декларативный_язык_программирования) и счётными машинами типа маткада.

Задача математика-теоретика в том, чтобы создавать то, что ты назвал "правилами вывода, формулами там всякими" и создавать с нуля теории. Задача математика-практика в том, чтобы отождествлять объекты реального мира с конструкциями, которые выдумали теоретики. А считают давным-давно роботы (в том числе биороботы, если вы понимаете, о чём я).
>> No.90838 Reply
>>90837
Спасибо! Камень с плеч упал.
Но теперь всплыла другая загадка, а какие процессы происходят вот в этом самом создании теорий с нуля. И какие сейчас животрепещущие нерешенные вопросы математики (как теоретической, так и практической)?

И можно чуть расшифровать "отождествление объектов реального мира с конструкциями, которые выдумали теоретики"? Ну, т.е. я понимаю, физика - что-то наблюдаем, потом пытаемся найти математический аппарат, который бы всё это описывал, а значит позволял моделировать и прогнозировать. А наоборот - это как?

З.ы. Пардон, если вопросы совсем тупые.
>> No.90844 Reply
>>90786
Друг, надо БОЛЬШЕ таких няш, а кайф от матана придет.
>> No.90863 Reply
>>90832
Сходится условно
>> No.90864 Reply
>>90863
Я понял, сам ряд изначальныйд условно. Но вот ряд an по какому признаку доказать, что он расх-ся?
>> No.90865 Reply
File: tumblr_lj7bflEuN8...
Jpg, 80.38 KB, 479×640
edit Find source with google Find source with iqdb
tumblr_lj7bflEuN81qanmz3o1_500.jpg
File: sGpX6VHzPQw.jpg
Jpg, 66.13 KB, 604×453
edit Find source with google Find source with iqdb
sGpX6VHzPQw.jpg

Вечер, матемач.
Порикаминдуй чего почитать по эллиптической криптографии для нубов. Вот няшка и котик.
>> No.90877 Reply
Хочу научиться ясно и четко доказывать. Сам всего лишь очередной недобыдлокодер, но часто не могу поверить в некоторые высказывания на слово и хочется их доказать. Но не могу это сделать с уверенностью, от этого расстраиваюсь и неуверенно сплю по ночам.
Т.е. хотелось бы развить навыки безотносительно какой-то конкретной теории. Насколько понимаю, надо ботать логику, конкретно, натуральную дедукцию. Верно? Пока что осилил fitch style и sequent calculus. Что ещё посоветуете?
Еще загорелся идеей написать простенький прувер для логик первого порядка, кто-нибудь здесь таким занимался?
>> No.90888 Reply
>>90864
Погоди, там логарифм в квадрате, или двоичный логарифм? Если логарифм в квадрате, то сходится абсолютно. Обычно для таких рядов используют сравнение с интегралом.
>> No.90896 Reply
File: kr9o_MWTy94.jpg
Jpg, 196.09 KB, 1260×1024
edit Find source with google Find source with iqdb
kr9o_MWTy94.jpg
File: 1342517736620.jpg
Jpg, 345.42 KB, 1280×850
edit Find source with google Find source with iqdb
1342517736620.jpg
File: BK4.jpg
Jpg, 1046.49 KB, 1748×1412
edit Find source with google Find source with iqdb
BK4.jpg

>> No.90936 Reply
>>90888
Просто ln2n, разобрался и все отлично. Спасибо, сдал. Но вот другое задание лютое зло. Спрошу позже, ибо реально уже 4 разных ответа давал, даже с вольфрама, но результат не устраивал. Как приеду отпишусь.
>> No.91013 Reply
File: 1360006055906.png
Png, 0.94 KB, 300×20 - Click the image to expand
edit Find source with google Find source with iqdb
1360006055906.png
>>90877
> Еще загорелся идеей написать простенький прувер для логик первого порядка
Плохо себе представляю принцип, но звучит потрясающе. По твоему лексикону могу судить, что ты на верном пути.
>> No.91079 Reply
>>90660
Большинству пока не интересна.
Недавно появилась homotopy type theory, которая может стать весьма популярной и полезной.
>> No.91106 Reply
>>91079
> и полезной
Для чего же?
Понятно, что речь идёт о верификации пруфов. Но, может быть, ты знаешь какие-то подробности?

Алсо, не проебал ли я трипкод
>> No.91109 Reply
Посоветуйте что почитать по терверу. Никак он мне не дается, постоянно все забываю.
Извиняюсь за тупой вопрос.
>> No.91117 Reply
File: 1343233243432.png
Png, 492.01 KB, 500×731 - Click the image to expand
edit Find source with google Find source with iqdb
1343233243432.png
Посоветуете учебник по логике?
>> No.91157 Reply
>>91117
Попробуй почитать созидателей мира сего: "введение в логику" Алонзо Черча. Сам дальше десятой страницы не смог, ты поймешь почему.
Алсо, гугли конкретную тему и читай методички на крайний случай, это гораздо легче чем заумные книжки ботать.
>> No.91162 Reply
>>91117
Было. Шенфилд Дж., "Математическая логика".
>> No.91163 Reply
Матемачан, есть вопрос. Чему равна производная от переменной величины по константе? Мне "на пальцах" кажется что 1, но никак объяснить это не получается.
>> No.91176 Reply
>>91163
Что такое производная по константе?
>> No.91178 Reply
File: be-rational-get-real_design.png
Png, 44.54 KB, 280×280 - Click the image to expand
edit Find source with google Find source with iqdb
be-rational-get-real_design.png
4 семестр, срочно нужна литература по комплексному анализу для тупых.
1)Хороший задачник с примерами решений некоторых номеров и обязательно ответа для проверки.
2)Учебник по теории.

какие знания требуются для предмета?
>> No.91179 Reply
>>91178
> Хороший задачник
Вот кстати, у Львовского просто талант к подбору качественных задач, смотри в конце его книжки.
Ответов там нет, спрашивай вопросы сюда, я иногда заглядываю.
>> No.91222 Reply
File: 1360238783847.png
Png, 0.84 KB, 300×20 - Click the image to expand
edit Find source with google Find source with iqdb
1360238783847.png
Учебник, прикреплённый в >>87532, для первоначального изучения предмета на уровне бакалавра средних вузов годится? Капча предлагает написать инбифо шёл бы ты, уг, толсто.
>> No.91223 Reply
>>91222
Обычных российских - нет. В НМУ своя, особая программа.
>> No.91224 Reply
>>91223
То есть я отстану от программы, что ли?
>> No.91225 Reply
>>91224
Просто изучишь разделы, которые в традиционных универах не трогают. Традиционные для отечественных универов линал, ангем и интегрирование по этой книжке не изучишь.
>> No.91226 Reply
File: слон.jpg
Jpg, 23.82 KB, 300×370 - Click the image to expand
edit Find source with google Find source with iqdb
слон.jpg
>>91225
Я же спрашивал только о предмете математический анализ. Так что с анализом, что с ним?
>> No.91227 Reply
>>91226
Анализ бывает очень разный. Эта книжка нетрадиционна, хочешь традиционный курс - читай Зорича или Демидовича.
>> No.91230 Reply
>>91222
Да, в этой книжке самый стандартный материал.
Но темп изложения там чуть выше общепринятого, и может быть немного сложновато при первом чтении.
>> No.91233 Reply
>>91230
Интеграл Лебега - стандартный? Лол.
>> No.91236 Reply
Начались численные методы, а я программировать, если честно, не умею. Это провал, да?
>> No.91237 Reply
>>91157
Пока неплохо идёт, спасибо.
>>91162
Пока не пробовал, спасибо.
>> No.91243 Reply
>>91236
Программирование там совсем простое, только формулы вбивать. Если совсем не умеешь, может быть, будет тяжело с непривычки, но, думаю, ничего страшного. Нужно знать, что такое массив, и уметь отладчиком пользоваться, вот, в общем-то, и всё.
>> No.91247 Reply
>>91243
Судя по всему нам можно будет пользоваться маткадом или на любом языке. Тогда попрошу литературу базовую по маткаду или статьи, а также интересный и простой для начала учебник по самим ЧМ.
Заранее спасибо.
>> No.91257 Reply
>> No.91270 Reply
>>91243
> что такое массив
Массив это монада.
>> No.91271 Reply
>>91270
Массив из n элементов - это n-элементный кортеж.
>> No.91272 Reply
>>91271
Вообще говоря нет. Кортеж в математике это кортеж в программировании.
>> No.91273 Reply
File: 779908954_5c10f0e12c_z.jpg
Jpg, 39.70 KB, 500×333 - Click the image to expand
edit Find source with google Find source with iqdb
779908954_5c10f0e12c_z.jpg
А правда ли, что в кольцах с делителями нуля можно делить на ноль?
И вообще, можно ли где-нибудь на него делить?
На проективной прямой, там, например?
>> No.91276 Reply
>>91272
Вообще говоря, под массивом в программировании можно понимать математическое понятие вектор.
>>91273
Нет. Делить на ноль нельзя. Но если при перемножении двух ненулевых элементов мы получаем ноль, то можно говорить о "делителях нуля" - это просто термин. Пример делителей нуля - матрицы ((1 0) (0 0)) и ((0 0) (0 1)).
>> No.91281 Reply
>>91276
> под массивом в программировании можно понимать математическое понятие вектор.
Какая глупость. Нельзя, конечно.
>> No.91283 Reply
>>91276
Нельзя. Вектор это скорее abstract datatype с заданным интерфейсом и с фиксированной статически длиной. Массив это что-то более низкоуровневое, но можно сказать что это упорядоченная последовательность однотипных элементов. (На самом деле это не совсем точно, ну да не важно.)
Более того само понятие массива несколько варьируется от системы типов к системе типов.
>> No.91284 Reply
>>91273
В расширенной комплексной плоскости делить на ноль можно. Это связано с тем, что в ней бесконечность — не предельно-недостижимое значение, а вполне конкретная точка, соответствующая точке (0, 0, 1) в стереографической проекции. Правда, при этом подобное множество внезапно перестает быть полем, но это мало кого волнует.
>> No.91286 Reply
>>91283
В информатике одномерные массивы часто называют векторами. Но математическое понятие всё же требует, пожалуй, операций. Это так, чтобы не пугались. нашли, о чём спорить, как обычно.
>> No.91288 Reply
>>91284
а если не на плоскости, а в каком-то проективном пространстве произвольной размерности?
>> No.91291 Reply
>>91283
Вектор - это более общее понятие. Все, что относится к массиву - можно выразить векторными выражениями.
Касательно фиксированности длины вектора. Твое замечание говорит о том, что массивы обладают неким свойством, которым векторы не обладают - это не так, и сейчас я поясню это:
Есть бесконечномерные векторные пространства - бери их элементы, проецируй на разные конечномерные пространства - и вуаля - у тебя массив переменной длины.
Итого получаем: массив - это реализация модели вектора. Возможно, эта реализация будет обладать дополнительными примочками, которыми напрямую вектор не обладает, но извернувшись (как я показал), их всегда можно получить.
>>91284
А вот с этим согласен.
>> No.91293 Reply
>>91291
> массив - это реализация модели вектора.
Вектор - это элемент векторного пространства. Массив не является элементом пространства, так как пространство массивов не определено, операции над массивами не заданы. Поэтому нельзя считать массив моделью вектора.

Массив из n элементов типа T - это элемент множества T^n, и ничего более.
>> No.91309 Reply
>>91247
Косарев В.И. 12 лекций по вычислительной математике - самое простое, что я видел.
>> No.91363 Reply
File: tn0-2.jpg
Jpg, 8.11 KB, 300×225 - Click the image to expand
edit Find source with google Find source with iqdb
tn0-2.jpg
Помогите разобраться.
Есть набор из 28 костяшек домино.
Нужно методами комбинаторики определить, сколько разных пар костей можно достать из набора, чтобы их можно было положить рядом друг с другом (чтобы они имели хотя бы одно общее число).

Определил, как без гугла унылого подбора подсчитать, какое максимальное количество точек содержится на одной стороне костяшки. (Хотя не факт, что это необходимо, лол.)

Собственно, не поможете ли?
А то уже второй день сижу над этой задачей.
>> No.91364 Reply
>>91363
У тебя есть 7 чисел, которые могут быть на костяшке(0-6), для каждого числа есть еще только 7 костяшек с хотя бы одним таким числом. То есть тебе нужно посчитать сколько пар можно составить из семи костяшек и умножить это число на семь:
7! / (2! (7 - 2)!) * 7 = 21 * 7 = 147.
>> No.91376 Reply
>>91364
Ооо...
Как же сильно я замудрил, однако.
Большое спасибо!
>> No.91387 Reply
Pocony, a kto na kakie speckursi khodit?
>> No.91388 Reply
>>91364

Как это семь числе на костяшке? У ней же шесть граней. Я вас не понимаю.
>> No.91389 Reply
>>91387

Ya hoshy na kursy gru po chelovechie-zmeechelovechey bor'be. Ochen interesno
>> No.91390 Reply
>>91388
A={0, 1, 2, 3, 4, 5, 6}. Всего 7.
>> No.91391 Reply
File: skmatn1011.pdf
Pdf, 0.23 KB, 841×595 - Click the image to get file
skmatn1011.pdf
>>91389
vibiraj eshe, ne zhalej sil!
>> No.91399 Reply
>>91388
Ты с кидательной чтоли перепутал?
На костяшке домино 2 числа на 1 грани.
>> No.91482 Reply
Аноны, а почему не любая конечная группа является циклической?
>> No.91499 Reply
>>91399
> Ты с кидательной чтоли перепутал?
Проиграл.
>> No.91507 Reply
>> No.91552 Reply
Вы в основном учитесь по методичкам и учебникам, выпускаемым вашим универом и лекциями или всегда стараетесь подкреплять другими источниками типа авторитетных книжек и программ других вузов? У меня специальность "прикладная математика и информатика", но математика какая-то слабая по всей видимости. Но может мне для программирования, что бы я не программировал, будет достаточно?
>> No.91554 Reply
>>91552
Вообще я теряю как-то ориентацию. Математики много. Всё не выучишь, все книги не прочтёшь. И дипломированность поди пойми, что означает.
>> No.91556 Reply
File: Lengyel-E.-Mathematics-for-3D-Game-Programming-and.pdf
Pdf, 8.39 KB, 252×323 - Click the image to get file
Lengyel-E.-Mathematics-for-3D-Game-Programming-and.pdf
>>91552
Релейтед - математика, нужная программистам.
>> No.91561 Reply
>>91399

Ну. Два снизу и два сверху. Если суммировать то два числа - верхнее и нижнее.
Если не суммировать четыре числа - два сверху, и два снизу.
А семь то откуда взялось?
>> No.91565 Reply
>>91556
Вот холодно ли мне без тензоров? И без офигенно правильного определения дифференциала, и без ZFC? Основания, понимаешь, наука, представление.
>> No.91566 Reply
File: url.jpeg
Jpeg, 134.04 KB, 900×900
edit Find source with google Find source with iqdb
url.jpeg
File: Шень-CS.pdf
Pdf, 1.07 KB, 363×272
Шень-CS.pdf

>>91552
> но математика какая-то слабая по всей видимости
Я представил, что ты выглядел как пикрелейтед, когда писал эту фразу, и решил утешить.

Во-первых, тысячи представителей люмпен-пролетариата прекрасно кадируют на джаве, пхп, си-шарпе и т.д., не зная ничего вообще кроме синтаксиса своего языка, и прекрасно живут.

Во-вторых, программистам высокого уровня нужно знать свои науки (которые принято относить к компутер саенс): логику, грамматики, теорию вычислимости, теорию алгоритмов, комбинаторику, численные методы, графы и т.д. При упорстве и трудолюбии ты можешь выучить какие-то основы этих наук дома по учебникам и задавая вопросы на англоязычных сайтах-для-вопросов, ничего принципиально сложного там нет.

Алсо отправляю доклад Шеня.
>> No.91567 Reply
>>91566
> кадируют
ох лол, опечатка пришлась как раз к месту
>> No.91571 Reply
>>91566
> ничего принципиально сложного там нет
А что вообще тогда есть «принципиально сложное»?
>> No.91576 Reply
>>91571
строгих критериев нет, кому-то и определение вещественных чисел понять принципиально сложно.

Разница вообще в том, что одни науки растут в глубину, а другие состоят из множества почти несвязанных друг с другом фактов (которые сами по себе могут быть технически весьма сложны).
>> No.91581 Reply
File: 29_38.jpg
Jpg, 44.33 KB, 346×500 - Click the image to expand
edit Find source with google Find source with iqdb
29_38.jpg
Ребят, вопрос на миллион. В чём прикладная польза иррациональных чисел? Ведь во всех прикладных науках и технике всегда используются рациональные приближения интересуеющих иррациональных чисел.

Что, ни в чём, кроме методической и научной целей?
>> No.91583 Reply
>>91581
Иррациональные числа - не выдуманный от балды конструкт, иррациональные числа возникают естественным путём при изучении рациональных чисел.
> В чём прикладная польза иррациональных чисел?
Они просто есть, и всё тут. Это как спросить, в чём прикладная польза смены дня и ночи.
>> No.91584 Reply
File: lHoEEoVG2tw.jpg
Jpg, 140.06 KB, 800×600 - Click the image to expand
edit Find source with google Find source with iqdb
lHoEEoVG2tw.jpg
>>90593
Аноны,помогайте.
Пишу диплом и запнулся в одном месте. Требуется обосновать количество проведенных опытов,при помощи коэффициента стьюдента,доверительного интервала и всего подобного,а я не совсем их понимаю. В гугле много статей,но они посвящены либо психологии,либо экономике, я теряюсь из за этого.
Объясните что к чему? С меня няша.
>> No.91586 Reply
>>91584
Это оценка погрешностей называется.
http://yadi.sk/d/UlT7nwzY0Vrck - с. 98, параграф 2.
http://www.himikatus.ru/art/modeling/ocwnka-errors.php - самая мякотка оценки погрешностей
http://www.sibupk.nsk.su/Public/Chairs/c_natural/MetLR/MetodLab.htm - подробнее.
ruwiki://Метод_наименьших_квадратов

физик
>> No.91587 Reply
File: 0a6279a85e5b.jpg
Jpg, 54.32 KB, 800×728 - Click the image to expand
edit Find source with google Find source with iqdb
0a6279a85e5b.jpg
>>91586
Ага,точно,так и называется. Плохо в этом ориентируюсь честно говоря.
Спасибо,сейчас почитаю.Надеюсь разберусь.
>> No.91589 Reply
>>91587
Не разберёшься - спрашивай ещё. :3
>> No.91590 Reply
>>91566
> люмпен-пролетариата прекрасно кадируют на джаве, пхп, си-шарпе
Какой ещё люмпен-пролетариат? Это всякие бичи и "операторы пека" - люмпены. А интерпрайз на джаве и пехопе - одна из самых высокооплачиваемых профессий вообще-то. Программист с хорошим опытом может рассчитывать на 150к в Рашке и в несколько раз больше в СШАшке.
>> No.91596 Reply
File: 107444031.jpg
Jpg, 143.17 KB, 523×785 - Click the image to expand
edit Find source with google Find source with iqdb
107444031.jpg
>>91507
Спасибо, няша, вот это я спорол, что называется.

Алсо, подскажите кто-нибудь, как искать обратные элементы в поле Галуа? Чего-то я не понимат.
>> No.91597 Reply
File: IKRzZ.jpg
Jpg, 120.75 KB, 800×549 - Click the image to expand
edit Find source with google Find source with iqdb
IKRzZ.jpg
>>91596
Чего-то я плохо сформулировал - хотет найти обратный по умножению элемент в GF(p^q).
>> No.91599 Reply
Анон, как тебя изменила математика?
Меня немного изменила квантовая механика и вещества.

Я учусь на прикладного математика. С низкого старта.
Так печально, многие из других вузов типа ВШЭ, или мехмата хорошо секут в математике. И как человеки они похитрее будут. А я на 2-м курсе. Вроде учусь-учусь, хожу на курсы в НМУ - там нихера не понимаю, ибо они для эдвансед, лучше учусь одногруппников, а всё равно как математик - говно.

Вот блеа, печаль.

И да, не могу определиться куда мне специализироваться:
Точно не Теория чисел/Дискретка/Алгебра/Теория групп(колец и т.п.)
Нравится всё гладкое, всякие волновые функции, метрики, топологии.

Скажи, математик-кун, куда идти, что бы всё-таки востребованным быть? (С++ знаю и Хаскель немного)

И да, я устал и заебался, а с мертвой точки не сдвигаюсь, не чувствую, что учусь.

p.s. пишу тут впервые за очень долгое время.
[spoiler]c: заслуга опыт [/spoiler]
>> No.91602 Reply
>>91590
Это не отменяет того, что на джаве, пхп, си-шарпе в основном программирует именно люмпен-пролетариат просто дорогой, но и то, дорогой он только с точки зрения того самого люмпена, ибо любой птушник, школьник, дцпшник вполне может справляться с этой задачей и высшее образование здесь не требуется.
>> No.91604 Reply
>>91602
Может быть, тебе просто нравится слово "люмпен" :3
> люмпен-пролетариат просто дорогой
Оксюморон же
> высшее образование здесь не требуется
Никак не относится к "люмпен/не люмпен". Доктор наук может быть люмпен-пролетарием
>> No.91608 Reply
>>91604
Термин люмпен-пролетариат может быть использован косвенным образом по отношению к людям, проявляющим психологию этого рода людей, но не являющимися ими в привычном понимании этого слова.
>> No.91612 Reply
>>91566
> логику, грамматики, теорию вычислимости, теорию алгоритмов, комбинаторику, численные методы, графы и т.д
Всегда думал, что эти разделы - тоже математика.
>>91576
> кому-то и определение вещественных чисел понять принципиально сложно
>>91581
LOL
>>91590
Я недавно с одним юристом разговаривал. Говорю ему, что на С# подрабатываю, а он говорит, что Вася(наш общий знакомый, назвал его имя) - мой коллега. Я у него спросил, а почему он коллега. А он ответил, что он ему недавно комп починил за деньги, программист значит.
>>91608
Чего-то тебе пригорело. Глупо думать, что вышка нужна уже только, чтобы быть нормальным человеком, а не ассоциироваться с люмпеном. Вот так и поступают на факультеты культурологии всякие ради образованности или потому, что надо.
Покормил.
>> No.91620 Reply
>>91612
> > логику, грамматики, теорию вычислимости, теорию алгоритмов, комбинаторику, численные методы, графы и т.д
> Всегда думал, что эти разделы - тоже математика.
Если понимать "математику" в широком смысле, то относятся (ещё их можно отнести к "theoretical computer science", которую иногда целиком причисляют к математике).

К "central core mathematics", т.е. тем наукам, которые в основном и изучают на факультетах математики, они не относятся (хотя некоторые разделы логики имеют весьма важные и нетривиальные приложения к теории чисел и алгебре).
>> No.91623 Reply
>>91602
> ибо любой птушник, школьник, дцпшник вполне может справляться с этой задачей и высшее образование здесь не требуется.
Если бы любой школьник, ПТУшник, дцпшник мог справиться с программированием, то получали ли бы программисты такие зарплаты, как думаешь? Чтобы программировать, нужно обладать особым складом ума. "Чистым математикам" программирование недоступно, например.
>> No.91625 Reply
>>91597
Конечное поле - группа по умножению => по теореме Лагранжа a^(q-1) = 1 для любого a /= 0 (q - порядок поля)
Значит обратный к a - это a^(q-2)
>> No.91626 Reply
>>90593
Если 0.(9)=1, то чему равно 0.(6)?
>> No.91627 Reply
>> No.91629 Reply
Q: чему равно 0.(6)?
A: 2/3

Неделя высшей математики на Доброчане.
>> No.91630 Reply
File: wN1TC40u3T8.jpg
Jpg, 13.05 KB, 397×270 - Click the image to expand
edit Find source with google Find source with iqdb
wN1TC40u3T8.jpg
>>91629
На более сложные вопросы тут не отвечают.
>> No.91631 Reply
>>91623
> то получали ли бы программисты такие зарплаты, как думаешь
Да. Почему бы и нет. А они (зарплаты программистов) тебе кажутся большими? Менеджер, которые вбивает таблицы в экселе получает больше чем сертифицированный жавакодер например.
>> No.91632 Reply
>>91630
покажи эти вопрсы
>> No.91634 Reply
File: Безымянный.png
Png, 19.14 KB, 681×278 - Click the image to expand
edit Find source with google Find source with iqdb
Безымянный.png
Тот самый ньюфаг по тфкп выходит на связь. Я вот пример понял что был выше в книжке, тупо раскрыл скобки распределил вынес. Пробую делать тут такое, попадаю на идиотизм, который не знаю как дальше преобразовать. Хочу как то догнать, чтобы не быть совсем дубом.

Или я не правильно понял метод решения?
>> No.91635 Reply
File: iphone-30.jpg
Jpg, 31.41 KB, 422×506 - Click the image to expand
edit Find source with google Find source with iqdb
iphone-30.jpg
>>90593
> 1.2. Свойства неопределенного интеграла.
> 1. ( f (x)dx) = f (x)
2. d ( f (x)dx)= f (x)dx ,
3. F(x)dx = F(x) + C или òd(F(x)) = F(x) + C ,
4. Af (x)dx = f (x)dx,
5. ( f (x) + g(x))dx = f (x)dx + g(x)dx .
Аноны-математики, поясните пожалуйста по хардкору что такое -d . Во второй строке.Это что знак дефферинциирования? Ответе пожалуйста. И если можно ссылочку на интересные статьи ,где для нубов объясняют что такое интеграл. Много ли в комьютерной графике применяют интегралы?
>> No.91637 Reply
>>91632
Расскажите уже мне как обращать в мультипликативной группе конечного поля! (не он, но все-таки)
>> No.91640 Reply
>>91637
выше кто-то написал же
>> No.91643 Reply
File: Sorry.+I+have+more+funny+than+cute+_a03ceab5cc38d1.gif
Gif, 346.46 KB, 300×151 - Click the image to expand
edit Find source with google Find source with iqdb
Sorry.+I+have+more+funny+than+cute+_a03ceab5cc38d1.gif
>>91640
Ой, в самом деле, спасибо. Но это же такая-то куча вычислений, там никаких фокусов нету?
>> No.91644 Reply
>>91643
Вроде нет. Возводить в большую степень можно быстрым алгоритмом enwiki://Exponentiation_by_squaring.

Другой метод нахождения обратных работает, если у тебя поле Галуа представлено, как F[x]/pn(x), pn(x) - неразложимый.
Тогда можно применить расширенный алгоритм Евклида в кольце F[x], и найти такие полиномы g(x) и h(x), что a(x)g(x) + p_n(x)h(x) = 1
Тогда g(x) - обратный для a(x) в F[x]/p_n(x). Но тут нужно делить полиномы с остатком, так что тоже куча вычислений.
>> No.91646 Reply
>>91631

Был менеджером, уверяю вас что всё в точности наоборот. А по части топ-менеджеров их надо сравнивать с сотниками программистов, которые сами когда то были програмистами, зарплата таких должностей обычно коммерческая тайна.
>> No.91653 Reply
>>91589
Спрашиваю еще. С погрешностями разобрался,спасибо. Теперь новый вопрос.
Мне нужно сделать измерения некой величины. Как мне узнать количество измерений которое требуется сделать? С достоверностью допустим 95%. В сети нашел онлайн-калькулятор который это делает,но меня смущает то что он для менеджеров-считает количество респондентов для опроса.Я понимаю что суть одна,но не доверяю. Да и вообще не понимаю как он работает.
>> No.91664 Reply
0,(9)+0,(0)1=1
>> No.91669 Reply
>>91664
Замени несуществующее 0,(0)1 на эпсилон и будет правильно. И не слушай здешних дурачков.
>> No.91670 Reply
>>91669
А мне в прошлом треде втирали, что такого числа нет! Вот и слушай людей после этого.
>> No.91673 Reply
>>91669
> Замени несуществующее 0,(0)1 на эпсилон и будет правильно
Не будет.
>> No.91675 Reply
File: 32164894.png
Png, 32.39 KB, 500×568 - Click the image to expand
edit Find source with google Find source with iqdb
32164894.png
>>91653
> Как мне узнать количество измерений которое требуется сделать? С достоверностью допустим 95%.
По таблице Стьюдента.

ruwiki://Процентили_распределения_Стьюдента
>> No.91679 Reply
File: 9vJ2IER6S0I.jpg
Jpg, 94.61 KB, 500×750 - Click the image to expand
edit Find source with google Find source with iqdb
9vJ2IER6S0I.jpg
Вечер, добрый.
Есть несколько диффуров, нужно решить до завтра. Пока штудирую литературу (http://решу.рф/математика/Филиппов/), но не особо помогает. Постоянно какие-то тупики. Собственно примеры:
1) Система x'=2x-y; y'=2y-x-5(e^t)sint
2) y'''' + y''=10x+13cosx (найти общее решение)
3) y'sqrt(x)=sqrt(y-x)+sqrt(x)
И да, в 3м думал про замену переменных, но если y=tx, тогда получается sqrt(t), что не хорошо.
С меня - что анон пожелает.
inb4: не твой личный решебник
>> No.91686 Reply
>>91679
1) и 2) - линейные неоднородные. Постарайся сначала сам решить.
3) Делаем замену y(x) = z(x) + x
>> No.91690 Reply
File: dem_eyes__by_rosemm25-d5rvx5b.jpg
Jpg, 102.39 KB, 900×602 - Click the image to expand
edit Find source with google Find source with iqdb
dem_eyes__by_rosemm25-d5rvx5b.jpg
Сап, котики.

А правда ли, что 0.(9)=1 ?
А то некоторые говорят, что чуть меньше.
Что же делать, что делать?!
>> No.91691 Reply
>>91690
Интуитивно предполагаю, что это свойство должно относится к квантовой физике.
>> No.91692 Reply
>>91690
Ну хоть ты-то душу не трави.
>> No.91693 Reply
>>91653
Ну это неправильная постановка вопроса. Для заданного количества измерений ты можешь с вероятностью 95% оценить интервал, в который попадает величина (95%-ный доверительный интервал). Чем больше измерений - тем больше точность оценки.

Пусть Xi = x + errori, где errori одинаково распределены со средним 0, x = EXi - величина, которую хотим найти.
Считаем среднее x' = sum(Xi) / n. Тогда из ЦПТ величина x'-x асимптотически распределена, как N(0, sigma^2 / n), где sigma^2 - дисперсия errori (неизвестная).
Эту дисперсию оцениваем, как D' = 1/n * sum(X_i - x')^2. [Можно использовать несмещённую оценку с 1/(n-1) вместо 1/n]

=> строим доверительный интервал x = [x' - Z(0.975) * sqrt(D'/n), x' + Z(0.975) * sqrt(D'/n)], где Z() - квантильная функция
нормального распределения. Z(0.975) ~ 1.96 (Здесь я использовал то, что она симметрична, поэтому Z(0.025) = -Z(0.975) ~ -1.96)

Можно предположить, что error_i нормально распределены, и использовать квантили распределения Стьюдента вместо нормальных, но, насколько я знаю, это давно не модно, сейчас принято использовать более общий подход (Стьюдент сходится к нормальному при увеличении количества степеней свободы, т.е. размера выборки в данном случае)
>> No.91697 Reply
>>91690
Да, эта правда. 0.(9) = 1, причём точно и именно в таком виде, без левого перехода к пределу.
>> No.91700 Reply
Няши, а какие крупные математические открытия сделаны в последнее время?
>> No.91703 Reply
File: dqIEYpBjpkA.jpg
Jpg, 100.06 KB, 900×589 - Click the image to expand
edit Find source with google Find source with iqdb
dqIEYpBjpkA.jpg
>>91693
>>91675
Так,примерно понятно. Меня смутило то что руководитель посмотрел на мои измерения и спросил,почему именно такое число опытов.В ответ на мой недоуменный взгляд он отправил меня курить коэффициенты Стьюдента и все то о чем вы говорите сейчас.Вообщем на сколько я понял - моя задача взять некое n количество измерений,посчитать для него всю эту фигню и опираясь на расчеты сказать что число опытов достаточно,или наоборот,не достаточно.
Спасибо,и добра вам всем!
>> No.91728 Reply
File: 7.jpg
Jpg, 296.04 KB, 2951×747 - Click the image to expand
edit Find source with google Find source with iqdb
7.jpg
Анон, я летчик в отношении линейных операторов. Один базис упоительнее другого. Помоги, молю.
>> No.91754 Reply
>>91700
Доказательство гипотезы Пуанкаре.
>> No.91785 Reply
>>91634
Чому никто не ответил :<
>> No.91786 Reply
File: Maple13_logo.jpg
Jpg, 22.04 KB, 193×46 - Click the image to expand
edit Find source with google Find source with iqdb
Maple13_logo.jpg
Кто-нибудь мне может объяснить как маплом пользоваться вообще? Не что писать и какой синтаксис, а что означают эти шизофренические режимы ввода: кнопки text и math раскрывающийся список со словами типа "math onput" и другими. Почему вставленная матрица имеет розовый цвет, если не введёшь значения? Почему он иногда при постановки после ее присвоения ";" не выводит матрицу, а выводит незамысловатое "matrix с какими-то цифрами"? Почему он иногда ничего не делает при нажатии на enter? Почему он иногда не видет, инициализированные выше переменные? Что это вообще за шизофрения?
>> No.91789 Reply
>>91754
А что ещё?
>> No.91796 Reply
File: Безымянный-рисунок.png
Png, 31.57 KB, 946×263 - Click the image to expand
edit Find source with google Find source with iqdb
Безымянный-рисунок.png
>>91786
Вот конкретно, почему он не выводит матрицу нормально?
>> No.91800 Reply
>>91796
Пардон, = на := я конечно догадался исправить, но результат идентичен.
>> No.91806 Reply
>>91670
Бесконечность тоже не существует как число. Сириусли, достаточно понять, что "сколь угодно большое число" и "сколь угодно малое" в определениях классического анализа - понятия абстрактные, и не надо даже приплетать сюда R+
>>91673
А вот и здешний дурачок.
>> No.91807 Reply
File: 1352654658635.jpg
Jpg, 314.57 KB, 1211×1014 - Click the image to expand
edit Find source with google Find source with iqdb
1352654658635.jpg
Аноны, доставьте годный учебник по теоретической механике, желательно самые-самые основы.
>> No.91809 Reply
>>91806
> А вот и здешний дурачок.
Обзываться любой дурак может. Ты лучше расскажи всё со строгим обоснованием, блесни мудростью.
>> No.91810 Reply
>> No.91811 Reply
>>91809
Обязательно. Всему свое время, сынок.
>> No.91812 Reply
File: 3308009006_065fe02894_z.jpg
Jpg, 96.48 KB, 640×426 - Click the image to expand
edit Find source with google Find source with iqdb
3308009006_065fe02894_z.jpg
>>91811
Отвечать за свои слова не хочет, зато самомнения полные штаны.
Всё как обычно.
>> No.91818 Reply
>>91728
Хочешь, чтобы тебе рассказали про линейные операторы?
>> No.91822 Reply
>>91812
Что за уродливый панк?
>> No.91823 Reply
>>91822

А Вил его знает!
>> No.91825 Reply
Всем привет, я волшебник. И принёс вам моё знание.

Действительное число - это
1) Значёк + или минус(+ опускают при записи)
2) Целое число, называемое целой частью
3) Бесконечная последовательность цифр, то есть отображение из N в {0,1,2,3,4,5,6,7,8,9}. Бесконечностей или епсилонов в N нету. Под отображением понимается однозначное сопостовление каждому элементу из N элемента из {0,1,2,3,4,5,6,7,8,9}
Всё что подходит под это определение является действительным числом, а то, что ему не удовлетворяет, действительным числом не является

Любое число вида m/n, где m и n(=/=0) - целые, имеет представление в виде периодической десятичной дроби(десятичные дроби с конечным хвостом считаются имеющими период 0). Любая периодическая десятичная дробь тоже имеет представление в виде m/n

0.(0) = 0
0.(1) = 1/9
0.(2) = 2/9
0.(3) = 1/3
0.(4) = 4/9
0.(5) = 5/9
0.(6) = 2/3
0.(7) = 7/9
0.(8) = 8/9
0.(9) = 1
>> No.91827 Reply
>>91822
Это не панк.
>> No.91834 Reply
File: 8NmKCS4dRLs.jpg
Jpg, 92.08 KB, 900×573 - Click the image to expand
edit Find source with google Find source with iqdb
8NmKCS4dRLs.jpg
>>91825
Максимально приемлемое и наглядное объяснение. Предлагаю выкинуть его в заголовок следующего треда.
>> No.91837 Reply
File: Аксиомы-сложения.JPG
Jpg, 45.33 KB, 920×167
edit Find source with google Find source with iqdb
Аксиомы-сложения.JPG
File: аксиомы-умножения...
Jpg, 50.28 KB, 908×195
edit Find source with google Find source with iqdb
аксиомы-умножения.JPG
File: Аксиомы-порядка.JPG
Jpg, 77.01 KB, 1254×252
edit Find source with google Find source with iqdb
Аксиомы-порядка.JPG
File: Аксиома-Дедекинда...
Jpg, 54.31 KB, 1253×114
edit Find source with google Find source with iqdb
Аксиома-Дедекинда.JPG

>>91825
> Действительное число - это
Не могу не запостить аксиоматическое определение.
>> No.91838 Reply
>>91837
Это вовсе не определение.
>> No.91839 Reply
>>91838
Определение. Множество действительных чисел - любое множество с этими аксиомами.
>> No.91840 Reply
>>91825
Всё равно я, как всегда, нехуя не понял. Почему я такой тупой?
>> No.91848 Reply
>>91840
Давай попробуем тебе помочь. Скажи мне, что ты собираешься понять.
>> No.91850 Reply
File: Разрыв-шаблона-начало.jpg
Jpg, 3.60 KB, 480×510 - Click the image to expand
edit Find source with google Find source with iqdb
Разрыв-шаблона-начало.jpg
>>91840
Сперва создадим тебе, мимокрокодилам и прочим слушателям интуитивное представление о бесконечностях с помощью простых вопросов и ответов. Я буду спрашивать, а вы - отвечайте. Аноны, уже владеющие матаном: немного помолчите, пожалуйста.

Вот картинка. На картинке два отрезка, первый и второй. Первый вопрос: в каком отрезке больше точек?
>> No.91859 Reply
>>91839
Здесь не сказано, как определены конкретные числа с конкретными записями. И 0.(9) пока не имеет смысла. Поэтому это не спасает отца демократии. (Есть чёткие определения +,* и <, с которыми моё более просто определение будет конечно удовлетворять всем этим аксиомам). По этим аксиомам они начнут записывать числа по своему. Когда математики пишут 0.(9), они свою собственную конкретную договорённость используют.
>> No.91875 Reply
>>91850
Очевидно, что точек бесконечно и там, и там. Но во во втором бесконечность больше, если так можно сказать. Всё правильно?
Мимокрокодил
>> No.91881 Reply
File: Разрыв-шаблона-конец.jpg
Jpg, 9.26 KB, 480×510 - Click the image to expand
edit Find source with google Find source with iqdb
Разрыв-шаблона-конец.jpg
>>91875
Нет, неправильно. Можно установить взаимно-однозначное соответствие между точками этих отрезков, например, как на картинке.
Во всех отрезках "одинаковое количество" точек. Более того, точек "одинаковое количество" в отрезке, в прямой, в плоскости и т.п.

Второй вопрос. Можно ли пронумеровать точки отрезка, то есть каждой точке отрезка поставить в соответствие одно-единственное натуральное число?
>> No.91890 Reply
File: 12723834135037.png
Png, 199.85 KB, 544×400 - Click the image to expand
edit Find source with google Find source with iqdb
12723834135037.png
>>91881
> Можно ли пронумеровать точки отрезка, то есть каждой точке отрезка поставить в соответствие одно-единственное натуральное число?
Должна ли нумерация идти последовательно?
>> No.91894 Reply
Главное, чтобы у каждой точки отрезка был натуральный номер, и чтобы номера не повторялись. Можно так сделать?
>> No.91895 Reply
>>91891
Вероятно да.
Мимокрокодил
>> No.91903 Reply
>>91894
Да, можно.
>> No.91904 Reply
>>91850
Одинаковое количество.
>>91881
Можно, только этих чисел будет бесконечное кол-во.
>> No.91911 Reply
А вот фиг вам, няши. Нельзя.

Так как во всех отрезках "одинаковое количество" точек, ограничимся рассмотрением отрезка от нуля до единицы и покажем, что его точки не пронумеровать.

Известно, что каждой бесконечной десятичной дроби вида 0,abcd... соответствует какая-то точка нашего отрезка (если не известно, скажите, я объясню, почему).

Допустим, мы пронумеровали точки отрезка, то есть каким-то образом поставили всякой бесконечной десятичной дроби в соответствие некоторое натуральное число. Теперь мы можем записать примерно такой список:

номер -> дробь
1 -> 0,57893565778356...
2 -> 0,24896590783...
3 -> 0,672008765...
4 -> 0,9271456732...
...
слева номер, справа некоторая бесконечная дробь, которую мы занумеровали этим числом.

Теперь сконструируем бесконечную дробь x следующим способом. Она будет начинаться на 0, чтобы принадлежать нашему отрезку. Пусть первая её цифра после запятой - любая цифра, но не такая, какая стоит на первом месте в числе №1, т.е. не 5 на нашем примере. Вторая её цифра будет не такой, какова вторая цифра числа №2, т.е. не 4 на нашем примере. Третья цифра - не 2 и т.д.
То есть построенная нами дробь x и дробь №k обязательно будут отличаться k-й цифрой после запятой (а другие цифры после запятой у них могут и совпасть).

С одной стороны, дроби x соответствует какая-то точка отрезка. С другой стороны, у дроби x нет номера, так как какой бы номер k мы не взяли, дробь x будет отличаться от дроби номер k.


Таким образом, точки отрезка нельзя занумеровать, поскольку всегда останется точка без номера. Выходит, хотя и натуральных чисел, и точек в отрезке бесконечность, точек в отрезке "больше", чем натуральных чисел. Это значит, что существуют разные бесконечности.
>> No.91912 Reply
Вопрос номер три. Пусть есть прямая. Возьмём на ней две несовпадающие точки. Всегда ли между ними можно поставить третью точку?
>> No.91913 Reply
>>91911
Ня, то есть существуют разные бесконечности по числу элементов. То есть бесконечности плоскости и прямой равны между собой и больше, чем множество натуральных чисел?
>> No.91914 Reply
>>91912
Да, всегда.
>> No.91917 Reply
>>91913
> То есть бесконечности плоскости и прямой равны между собой и больше, чем множество натуральных чисел?
Равны не сами бесконечные множества, а их так называемые "мощности".
> то есть существуют разные бесконечности по числу элементов
Правильнее говорить, что существуют бесконечные множества разных мощностей.
Точки прямой и плоскости можно поставить во взаимно-однозначное соответствие - множество точек прямой и множество точек плоскости равномощны. Натуральные числа и точки отрезка нельзя поставить во взаимно-однозначное соответствие - множество натуральных чисел и множество точек отрезка неравномощны.
Множества, равномощные множеству натуральных чисел, называют "счётными", а множества, равномощные отрезку от нуля до единицы называют "континуальными".
>> No.91918 Reply
>>91914
Верно, ня.

Вопрос четыре. Квадраты - это числа 1, 4, 9, 16 и т.д., то есть числа, которые являются квадратами натуральных чисел. Чего больше: натуральных чисел или квадратов?
>> No.91922 Reply
>>91917
> Множества, равномощные множеству натуральных чисел, называют "счётными", а множества, равномощные отрезку от нуля до единицы называют "континуальными".
Все множества либо счётные, либо континуальные?
>>91918
> Чего больше: натуральных чисел или квадратов?
Равное количество. Ведь я могу сопоставить любому числу его квадрат, а квадрату - число.
>> No.91929 Reply
>>91922
> Все множества либо счётные, либо континуальные?
Нет, существуют и другие бесконечные мощности, но для практических применений обычно хватает этих двух. Интересный факт: Кантор, основатель современной теории множеств, получил психоз, пытаясь доказать гипотезу о существовании мощности, промежуточной между счётностью и континуумом. Позднее было показано, что утверждение о существовании/несуществовании такой мощности недоказуемо в аксиоматике, которой пользовался Кантор, и должно вводиться как аксиома.

Кроме бесконечных мощностей, существуют также и конечные мощности. Мощность конечного множества - это всего-навсего число элементов в нём.
> Равное количество.
Верно. И множество квадратов, и множество натуральных чисел - счётные множества.

Я ухожу спать, вернусь завтра. Всем хороших снов.
>> No.91955 Reply
> Можно установить взаимно-однозначное
Если есть отрезки [a,b] и [c,d] на одной оси, то однозначное соответствие такое:
x2 = ((c-d)/(b-a))(x1-a)+c
Кстати между (-inf;+inf) и (a,b) такое:
x2 = ((b-a)/2)(2/п)arctg(x1) + (b-a)/2 + a
Сокращения не сделаны, чтобы сохранить логику построения.
> Второй вопрос. Можно ли пронумеровать точки отрезка, то есть каждой точке отрезка поставить в соответствие одно-единственное натуральное число?
Ты задал вопрос неправильно. Ты должен был уточнить, что соответствие должно быть взаимно однозначным. То есть, каждая точка должна иметь уникальный номер, и наоборот у каждого номера должна быть своя точка.
>>91929
Если я ничё не путаю, множество составленное из всех подмножеств данного множества всегда имеет большую по сравнению с исходным мощность. Так что мощность бывает сколь угодно большая.
>> No.91959 Reply
>>91955
только вместо с-d - d-c
>> No.91962 Reply
File: oh-you-93067263235.jpeg
Jpeg, 21.22 KB, 447×335 - Click the image to expand
edit Find source with google Find source with iqdb
oh-you-93067263235.jpeg
>>91929
> практических применений
> наивная теория множеств
>> No.91964 Reply
>>91962
А нельзя вместо выражения брюзгливости, логически высказывать свою позицию?
>> No.91997 Reply
Я слишком тупой и я даже завалил домашку первую же на курсере по математика.
Я нихера так и не понял про область определения и обратные функции, в первой же видеолекции об этом ни слова не было. Я думал там с нуля надо.

Как жить то, как жить то? YouTube: Как жить то бл*?!!!

Что читать, как быть.

Кто еще проходил у них курс одной переменной?
>> No.92001 Reply
>>91997
> даже завалил домашку первую же на курсере по математика
На математике в универе совершенно не действует правило "чем дальше тем сложнее". Нам на матане в первый день даи задания на построение графиков функций. Я эти кардиоиды с лемнискатами и прочие экзотичные кривые и сейчас на 3-ем курсе не осне умею изображать, умею конечно, но медленно и с большим использованием инструментов типа производных.
>> No.92002 Reply
>>91997
Множество - это совокупность объектов. Например, X = {яблоко, груша}, Y = {2,5}.
Отображение f:X->Y - это правило, по которому каждому элементу множества X ставится в соответствие некоторый элемент множества Y. Например, яблоко -> 2, груша -> 5. Отображение, если не вдаваться в детали, то же самое, что функция. Говорят, что X - область определения f. Также говорят, что f ставит в соответствие объекту x из множества X объект f(x) из множества Y.

Пусть есть отображение f:X->Y. Функция, которая по известному f(x) доставляет x, называется функцией, обратной к f. Например, пусть g - обратная к f. g(2) = яблоко, g(5) = груша.

Основное свойство обратной функции: g(f(x)) = x.
>> No.92006 Reply
>>91997
А что это такое? Что за курсер по математика? У кого у них?

Про область, чтобы понимать, надо понимать, что функция по определению - это, внимание, тройка - два множества и закон, по которому каждому элементу из одного множества ставится в соответствие один элемент из второго. Вот первое из них - это область определения.
Функции, перечисленные ниже являются совершенно разными:
f:[-п/2;п/2] -> [-1;1], f=sin x - биективная, имеет обратную
f:[-п/2;п/2] -> R, f=sin x - иньективная, имеет только левую обратную
f:R -> [-1;1], f=sin x - сюрьективная, имеет только правую обратную
f:R -> R, f=sin x - ни иньективностью, ни сюрьективностью не обладает.
f:{15;1;789;32768;-997} -> [-1;500], f=sin x - ни иньективностью, ни сюрьективностью не обладает. Именно в матане(не математике вообще) такую фигню не рассматривают обычно. Она там не интересна обычно. В матане любят функции, у которых множества с хорошими свойствами. А вот в комбинаторике когда ищут, например, способы разкладывания 7-ми пронумерованных книг по 10-ти пронумерованым полкам, говорят, что мпособ - это функция из {1,2,3,4,5,6,7} в {1,2,3,4,5,6,7,8,9,10}
Я уже не понимаю, зачем я учусь на специальности yoba-прикладная математика и информатика, если какую усвоенную лекцию я не перескажу, набегу топологи будут орать, что это лажа и всё не то.
>> No.92007 Reply
>>92002
> Отображение, если не вдаваться в детали, то же самое, что функция
Какие такие детали? Отображение, функция и оператор суть синонимы.
>> No.92041 Reply
File: zada.PNG
Png, 38.91 KB, 606×968 - Click the image to expand
edit Find source with google Find source with iqdb
zada.PNG
>>92006
>>92002
> Множество - это совокупность объектов. Например, X = {яблоко, груша}, Y = {2,5}.
Да это я знаю. Я даже умею шок брать частные производные, ряды сходимости исследовать и т.д.
Но вот элементарные теоретические вопросы я вот не знал.

Вот такие задания были, только не говорите ответы на нынешние, прошу, хочу заново пройти.
какое же я ничтожество.
>> No.92052 Reply
>>92007
Внезапно это не так. Кроме полных функциональных отображений, которые и называют кратко функциями, бывают и другие отображения.
>> No.92054 Reply
>>92007
У нас говорят функция когда множество значений находится в R, отображение в остальных случаях. Но над правильностью использования никто не трясется.
>> No.92112 Reply
Добровечер, аноны.
Есть одна задача - хочу подогнать под набор точек некоторое гомеоморфное сфере многообразие (вроде правильно сформулировал). Как к ней подступиться, чего почитать, какие похожие задачи были в истории?
сырбор петр
>> No.92115 Reply
>>92112
сформулируй строго, что требуется
и зачем это вообще надо?
>> No.92139 Reply
>>92115
Есть неизвестное (за исключением конечного набора точек), непрерывное преобразование сферы. Нужно построить его гладкое приближение. Я сейчас даже не понимаю, среди каких функций искать решение. Что будет обобщением полинома на сферу?
Прикладная задача аппроксимации
>> No.92143 Reply
>>92139
я всё ещё не понимаю, щито ты имеешь ввиду
> непрерывное преобразование
т.е. сфера гомеоморфно отображается на себя, какие-то точки куда-то уезжают и нужно придумать такой диффеомормизм сферы в себя, чтобы в ограничении на эти точки он совпадал с изначальным отображением? (а остальные точки могут делать что угодно?)
>> No.92146 Reply
>>92143
Сфера непрерывно отображается в эрэн. Есть такие-то точки на сфере и есть их образы. Нужно вписать в эти точки гладкое отображение сферы. Остальные точки пусть делают чего хотят, из всех решений мы потом выберем такие, которые выглядят нестрашно.
Короче говоря - та же интерполяция полиномами, которую нужно как-то обобщить на сферу.
>> No.92148 Reply
>>92146
Ну ок, что заказывали, то и получите.

Берём точку сферы, которая не является выделенной, делаем из неё стереографическую проекцию. При этом все точки кроме центра проекции переходят в эрэн (размерности сферы). Там уже интерполируем многочленами каждую координатную функцию в отдельности (в данном случае координатными функциями станут композиции обратного к проекции и данного изначально непрерыной координатной функции). Дальше берём функции, которые экспоненциально и бесконечно гладко зануляются вне некоторой окрестности, а внутри некоторой меньшей окрестности равны единичке (их нетрудно построить ручками) и домножаем каждую координатную функцию в окрестностях каждой точки на такую. В итоге получается, что почти всё эрэн отображается в начало координат другого эрэна, а заданные точки отображаются куда надо. Разумеется, центр проекции тоже отображаем в начало координат. В итоге получаем гладкое отображение с нужными свойствами.

Для всякой прикладноты, я уверен, существуют более разумные методы, но мне они не знакомы, увы.
>> No.92149 Reply
Доброкафедра, известно, что множество строк с операцией конкатенации (объединения) является моноидом: конкатенация ассоциативна (a + (b + c)) = ((a + b) + c), а нейтральным элементом является пустая строка "".

А что если ввести для каждой строки обратную? Например, для "Тарас" обратной будет -"Тарас"
"Тарас" + -"Тарас" = ""; 
"Тарас Бульба" - " Бульба" = "Тарас"
-"Тарас " + "Тарас Бульба" = "Бульба"
то есть сложение строки s = u+v со строкой -v даст u;
(u+v) + -v = u Тут возникает вопрос, как определить конкатенацию s + -v, если строка s не оканчивается на подстроку v. Можно определить это как s -v = vs; -v + s = sv
" Бульба" -"Тарас" = "Тарас Бульба"
-" Бульба" + "Тарас" = "Тарас Бульба"
Будет ли получившееся множество с расширенной конкатенацией группой?
>> No.92151 Reply
>>92149
Нет, ассоциативность теряется.
Б + (-Т + -Б) =  Б + Б + -Т = Б + ТБ = БТБ
(Б + -Т) + -Б = ТБ + -Б = Т
К тому же подозреваю, что так определённое сложение приведёт к противоречиям.
Но идея с обратными строками интересная.
>> No.92164 Reply
>>92148
Прикольная идея, мне все пригодится. Посоветуй еще пожалуйста какого-нибудь релейтеда читнуть.
>> No.92166 Reply
>>91786
>>91796
>>91800
Почти со всем разобрался. Я пишу на мапле численный метод решения СЛАУ. И мне удобно задавать входные данные матрицей A и вектором b(СЛАУ принимает вид Ax=b). Так вот, матрицы у меня правильно выводятся, если не писать restart. Убрал рестарт - работает. Не убрал - выводит вот это _rtable с цифрами. Как этого избежать и зачем вообще писать restart? Алсо, препод говорит, что лучше вообще перейти на classic-версию мапла. Так же поступают и другие студенты. Но ведь так вводить данные куда удобнее! Как быть? Есть какие-нибудь книжки по работе с маплам, не просто описывающие его возможности, а обучающие практике его использования?
>> No.92172 Reply
>>92164
не могу ничего посоветовать, никогда не имел дела с такими вещами
>> No.92179 Reply
>> No.92200 Reply
>> No.92222 Reply
File: 48a583d4dbdef2505b814ee3293ba299.jpg
Jpg, 128.97 KB, 738×1400 - Click the image to expand
edit Find source with google Find source with iqdb
48a583d4dbdef2505b814ee3293ba299.jpg
>>90593
Мне нравится, но я не понимаю математику. Пожалуйста, объясните в чём её суть? За что ухватиться надо, чтобы чувствовать её?
>> No.92231 Reply
File: 13603348288865.jpg
Jpg, 306.27 KB, 864×1291 - Click the image to expand
edit Find source with google Find source with iqdb
13603348288865.jpg
>>92222
Читай фихтенгольца.
>> No.92239 Reply
>>92231
Ещё будут советы?
>> No.92243 Reply
>>92222
В этом треде сидят одни дураки и пропагандируют идиотскую математику. Беги, без оглядки, быстро! Может быть сохранишь моск и когда-нибудь тебя потянет на "матан".
>> No.92244 Reply
>>92243
Как тебя понимать?
>> No.92246 Reply
>>92244
Это плохой тред, не спрашивай тут советов.

Лучше забацай тему в /b/ с какими-нибудь интересными парадоксами - я думаю народу много набежит и, если повезёт, развяжется воодушевляющий холивар.
>> No.92247 Reply
> 117 новых ответов последний: 18 February 2013
>> No.92248 Reply
File: verbitsky_msu.png
Png, 503.11 KB, 1098×400 - Click the image to expand
edit Find source with google Find source with iqdb
verbitsky_msu.png
>> No.92259 Reply
>>92222
> Пожалуйста, объясните в чём её суть?
В том же, в чём суть программирования или игры на музыкальном инструменте. Ты делаешь то, что нравится, и получаешь от этого удовольствие.
> За что ухватиться надо, чтобы чувствовать её?
Понять, что такое доказательство, и доказать что-нибудь самостоятельно.
>> No.92299 Reply
Анон, помоги решить задачу
Нет, это не только "ололо, я нихуя не делал весь семестр, решите мне", мне правда интересно как тут надо думать и что делать

при каком соотношении между коэффициентами a,b,c кривая y=ax^2+bx+c касается оси абсцисс
>> No.92300 Reply
>>92299
Если дискриминант больше или равен нулю, график функции пересекает ось абсцисс в двух точках. Когда дискриминант равен нулю, корни уравнения совпадают, то есть график касается оси абсцисс.
>> No.92305 Reply
>>92222
Суть программирования в том, чтобы заставлять компьютер выполнять полезную работы по ранению, обработке и передаче информации. Суть игры на музыкальном инструменте в создании и воспроизведении звукового художественного произведения, имеющего целью эстетическое наслаждение. Иногда, правда, музыка также несёт дополнительную функцию задания ритма(например, для танцев). Суть прикладной математики в решении задач, возникающих в разнообразной деятельности как то технические, естественные и общественные науки, программирование, управление и т.д., которые можно сформулировать на математическом языке, основанном на мат. логике.
>> No.92317 Reply
File: Снимок.PNG
Png, 169.57 KB, 923×1084 - Click the image to expand
edit Find source with google Find source with iqdb
Снимок.PNG
Я так понял авторы принимали тяжелые наркотики, если они тут ошиблись, то я так понимаю в уравнение по собственным числам тоже ошибка или нет?
>> No.92320 Reply
>>92317
Скорее всего, обведённое красным - просто опечатка. В характеристическом уравнении, уравнении tr A и det A на слайде 7 ошибки нет.
>> No.92325 Reply
>>92305
Я так раздразился, потому что ты мне поясняешь по остальным предметам будто являюсь умственно отсталым, что хотел тебе кожу с лица содрать. Но вспомнив, что понял суть математики, более не желаю этих фантастических переживаний. Спасибо за пост!
>> No.92332 Reply
>>92320
Благодарю, осталось разобраться с тем как изобразить траекторию что у меня в задании. Надеюсь к завтрашнему дню успею :3
>> No.92333 Reply
>>92325
> понял суть математики
и в чём же она?
>> No.92337 Reply
>>92333
Смысл математики это 42      
>> No.92338 Reply
>>92333
e^iPi=1
>> No.92341 Reply
>>92337
смысл математики это любовь
>> No.92402 Reply
Матан-куны, что думаете об этой статье: http://nbspace.ru/math/
>> No.92404 Reply
>>92338
e^(pi*i) = -1
>> No.92405 Reply
>>92402
Лажа полная. Математика - это не столько искусство, сколько инструмент, потому общее (среднее, да и высшее) образование подразумевает овладевание формальным инструментом с самой формальной точки зрения из возможных.
"Хочете" математики, как искусства - угорайте по факультативным спецкурсам сами.
>> No.92409 Reply
>>92338
EPIC FAIL
>> No.92410 Reply
>>92402
Да, всё так.
Это, вообще говоря, не какие-то откровения, а общее место.
>> No.92411 Reply
В статье очень много болтовни, да и фразочки про искусство мне не нравятся.
Под общим место я имел ввиду ублюдочность школьной программы (да и многих "мат"-школьных тоже)
>> No.92416 Reply
>>92405
Сам ты инструмент. От школьной математики пользы нет, кстати.

Статью читала давно, перечитывать лень.
>> No.92420 Reply
>>92402
Повсеместного засилья математики сейчас нет, и даже идёт процесс сокращения её присутствия. Из школьного курса исключили ряды, из ЕГЭ - интегрирование и производную. Это не так уж и печально, если вспомнить, что обычному человеку-телезрителю не требуется ни одна из наук - ни физика, ни химия, ни математика, ни история. Тем не менее, это влияет на общество негативно и приводит к иррациональному страху перед математикой.
В эссе верно подмечено, что в школах не занимаются математикой. Это действительно так, в школах на смену математике пришла "вступительная" математика, подробно про которую рассказано вот здесь: http://www.mccme.ru/edu/index.php?ikey=neretin В высших учебных заведениях математику тоже не преподают, за исключением необходимого всякому человеку с высшим образованием минимума, достаточного, чтобы описать возникшую перед ним проблему на языке, позволяющим пустить в дело калькулятор.
Бредовость современного образования (а она имеет место, да) объясняется тем, что в конце девятнадцатого века ведущие математики взяли на вооружение аксиоматический метод и создали ему такую рекламу, что во всех странах прошли реформы математического образования. Сейчас идут обратные процессы.
Эссе затрагивает проблему несовершенства мира. В школах страдают фигнёй, люди не хотят получать от жизни удовольствие, а хотят страдать... Так было и так будет. Мечтать хорошо, но живём-то мы в реальности.
>> No.92488 Reply
>>92420
Не привело бы это к нарастанию лудитских настроений. Щас как начнут орать:"Учёные дураки!", "Они могут быть не правы!", "А у меня своя истина!", "ГМО - яд!", "Эксперементы на людях!", "Ничё они не знаю, всё это глупость!" и т.д.
>> No.92511 Reply
Анон, это нормально три уравнения решать целый год в виде одного отдельного курса? Да так, что они с громким скрипом даются всем без исключения студентам? Уравнения боли и унижений математической физики, тупо три штуки: ΔU = f, Ut = ΔU + f и Utt = ΔU + f. Это мы такие реально тупые, как нам кажется?
>> No.92514 Reply
>>92416
Зависит. Кассирам в KFC, разумеется, нет - ведь под рукой калькулятор.
А мне на днях, когда от скуки в голову пришла идея посчитать (1+1/16+1/81+...+1/(n^4)+...) очень пригодились именно школьные знания, куда, в частности, входят простейшие операции над комплексными числами.
Никакого искусства и изящества в этом упражнении нет, например.
>> No.92515 Reply
>>92511
Проблема же не столько в решении уравнений, сколько в познании методов их решения.
В частности, если память мне не изменяет, подавляющей части населения требуется пол семестра на понимание сути разложения в Фурье.
>> No.92516 Reply
>>92514
Следует отметить, что я был в курсе, как брал в свое время Эйлер сумму обратных квадратов. Знал я это со школы. Те, кому это никогда не будет нужно этого тогда не узнали, и, думаю, не жалеют.
Я узнал и вывел по аналогии. Денег я за это не получил, но пригодилось это мне вполне.
>> No.92520 Reply
File: 0Yf9SPwi8ps.jpg
Jpg, 44.81 KB, 494×468 - Click the image to expand
edit Find source with google Find source with iqdb
0Yf9SPwi8ps.jpg
Пацаны, я не понимаю половину шуток с xkcd. Это сильно плохо?
>> No.92594 Reply
File: Nagomi.Hiyori.full.802761.jpg
Jpg, 222.68 KB, 800×569 - Click the image to expand
edit Find source with google Find source with iqdb
Nagomi.Hiyori.full.802761.jpg
>>90593
Анончик, упрости, пожалуйста. Сейчас занимаюсь математикой, учу по книге и решаю задания, а вот тут застрял. Поясни, как решить, не получается уже 2 день не решу.

((a-3)/(a^2-3a+9)-(2a-6)/(a^3+27))*((2a^3+54)/(a+1))
>> No.92604 Reply
>>92520
> Весёлая игра: найти комбинацию двух продуктов, которая больше всего встревожит кассира. Победитель: тест на беременность и плечики для куртки.
Меня это прямо заинтересовало. А в чём соль?
>> No.92605 Reply
>>92594
(3-а)*2/(а+1). Решал устно, извините за неровный почерк.
>> No.92606 Reply
>> No.92608 Reply
>>92605
Вольфрам не согласен...
>> No.92609 Reply
File: eurekaseven_talho1600.png
Png, 1196.68 KB, 1600×1200 - Click the image to expand
edit Find source with google Find source with iqdb
eurekaseven_talho1600.png
>>92605
Няша, распиши, все действия нужны.
>> No.92614 Reply
>>92594
((a-3)/(a^2-3a+9)-(2a-6)/(a^3+27))*((2a^3+54)/(a+1)) = 
2((a-3)(a^3+27) - (2a-6)(a^2-3a+9))(a^3+27) / (a^2-3a+9)(a^3+27)(a+1) = 
2((a-3)(a^3+27) - (2a-6)(a^2-3a+9)) / (a^2-3a+9)(a+1) = 
2((a-3)(a^3+27) - 2(a-3)(a^2-3a+9)) / (a^2-3a+9)(a+1) = 
2(a-3)((a^3+27) - 2(a^2-3a+9)) / (a^2-3a+9)(a+1) = 
2(a-3)(a^3+27 - 2a^2 + 6a - 9)) / (a^2-3a+9)(a+1) = 
2(a-3)(a^3 - 2a^2 + 6a +18)) / (a^2-3a+9)(a+1) = 
2(a-3)(a^3 - 2a^2 + 6a +18)) / (a^3-3a^2+9a+a^2 - 3a + 9) =
2(a-3)(a^3 - 2a^2 + 6a +18)) / (a^3-2a^2+6a+ 18) =
2(a-3)
>> No.92617 Reply
Почему до меня сразу не дошло?
>> No.92621 Reply
File: 1361577065354.png
Png, 1.36 KB, 300×20 - Click the image to expand
edit Find source with google Find source with iqdb
1361577065354.png
>>92617
Капча говорит, что молиться чаще надо. Иронизирует, наверное.
>> No.92622 Reply
File: gif.latex.gif
Gif, 7.39 KB, 360×309 - Click the image to expand
edit Find source with google Find source with iqdb
gif.latex.gif
>>92594
На пике
>>92606
Скобку забыл
http://www.wolframalpha.com/input/?i=%28%28a-3%29%2F%28a^2-3a%2B9%29-%282a-6%29%2F%28a^3%2B27%29%29*%28%282a^3%2B54%29%2F%28a%2B1%29%29
>>92614
Подгонщик :3
Шестая строка:
2(a-3)(a^3+27 - 2a^2 + 6a - 18)) / (a^2-3a+9)(a+1)
Предпоследняя:
2(a-3)(a^3 - 2a^2 + 6a +18)) / (a^3-2a^2+6a+ 9)
>> No.92623 Reply
>>92622
Я просто заблудился в копипасте.
((a-3)/(a^2-3a+9)-(2a-6)/(a^3+27))*((2a^3+54)/(a+1)) = 
2((a-3)(a^3+27) - (2a-6)(a^2-3a+9))(a^3+27) / (a^2-3a+9)(a^3+27)(a+1) = 
2((a-3)(a^3+27) - (2a-6)(a^2-3a+9)) / (a^2-3a+9)(a+1) = 
2((a-3)(a^3+27) - 2(a-3)(a^2-3a+9)) / (a^2-3a+9)(a+1) =   (тут была ошибка)
2(a-3)((a^3+27) - 2(a^2-3a+9)) / (a^2-3a+9)(a+1) =        
2(a-3)(a^3+27 - 2a^2 + 6a - 18)) / (a^2-3a+9)(a+1) =      
2(a-3)(a^3 - 2a^2 + 6a +9)) / (a^2-3a+9)(a+1) =           
2(a-3)(a^3 - 2a^2 + 6a +9)) / (a^3-3a^2+9a+a^2 - 3a + 9) =
2(a-3)(a^3 - 2a^2 + 6a +9)) / (a^3-2a^2+6a+9) =           
2(a-3)
>> No.92637 Reply
>>92622
Идеальный вариант. Спасибо! Видимо, мне нужно поработвать над разложением на множители, потому что именно из-за этого застрял.
>> No.92646 Reply
>>92622
Как вы до этого доходите? Что заставляет вас видеть эти ходы?
>> No.92651 Reply
File: Горнер.png
Png, 9.72 KB, 914×132
edit Find source with google Find source with iqdb
Горнер.png
File: перемножение-матр...
Png, 10.16 KB, 635×155
edit Find source with google Find source with iqdb
перемножение-матриц.png

>>92637
Здесь везде речь о многочленах одной переменной x.

Многочлен степени n(самая большая встречающаяся степень) всегда имеет ровно n корней, правда некоторые из них могут быть комплексными, некоторые - кратными. Кратные корни - это типа несколько одинаковых. Более правильно говорят, что у корня многочлена есть кратность. Например, если у многочлена второй степени дискриминант = 0, то это два кратных корня, а более правильно, корень с кратностью два. А если дискриминант >0, то это два корня кратности один. Более правильно сказать не n корней, а сумма кратностей всех корней равна n. Если дискриминант меньше нуля, то на самом деле корни есть, но они "чисто комплексные". Комплексные числа - это обобщение действительных, в этом смысле, если не вдаваться, все действительные числа - частные случаи комплексных. А те комплексные, которые действительными не являются называют чисто комплексными. Любой многочлен степени n разлагается на множители так:
a(x-x[1])^k[1](x-x[2])^k[2]...(x-x[m])^k[m]
здесь a - коэффициент стоящий при наибольшей степени x. x[] - корни многочлена. k[] - кратность соответствующего корня. Эта формула имеет место хоть для комплексных чисел, хоть для действительных. Конечно воспользоваться ей удастся, не имея дел с комплексными числами, только тогда, когда все корни многочлена окажутся действительными. Но даже если будут комплексные корни, если коэффициенты в многочлене были все действительные, то в любом случае можно любой многочлен разложить в такой вид:
a(x-x[1])^k[1](x-x[2])^k[2]...(x-x[m])^k[m](x^2+p[1]x+q[1])^l[1](x^2+p[2]x+q[2])^l[2]...(x^2+p[r]*x+q[r])^l[r]

Если вид комплексных чисел тебя не пугает, скажу, что в случае с многочленом с действительными коэффициентами эти все чисто комплексные корни будут иметь сопряжённую пару. Короче если u+vi кратности l - корень, то и u-vi - тоже корень кратности l. Каждый из этих квадратных множителей со степенью l соответствует паре таких сопряжённых друг другу комплексных корней, имеющих кратность l каждый. Причём p=-2u, q=u^2+v^2.

Зная корни, коэффициенты можно вычислить формулами Виета:
ruwiki://Формулы_Виета

ruwiki://Схема_Горнера
Схема Горнера позволяет гораздо быстрее вычислять значение многочлена при каком-то х. Это полезно для программирования такого вычисления и для быстрой проверки, является ли число корнем. Если многочлен имеет целые коэффициенты, а его корень представляется как дробь с целыми числителем и знаменателем(то есть, корень рационален, не иррационален), то числитель будет делителем свободного коэффициента(без х), а знаменатель - делителем старшего коэффициента(пере самой большой степенью). Тут нужно учесть, что 1 - делитель для любого числа, и само число всегда является делителем самого себя. Вывод: если надо посчитать все рациональные корни многочлена с целыми коэффициентами - выписываем все подходящие дроби, с подходящими числителями и знаменателями и поочерёдно проверяем их схемой Горнера. В ручной схеме Горнера, как её в ручную пишут, одновременно выписываются коэффициенты, которые остаются в многочлене после вынесения из многочлена множителя (х-корень) - см. пикрелейтед. - это типа побочный эффект.

С поиском всех корней многочленов степени не выше 2 всё и так понятно, не забываем, что квадратные уравнения решаются по формуле или формулами Виета. Корни многочленов -ей степени находятся методом Кардано, а 4-ой - методом Феррари
ruwiki://Формула_Кардано
ruwiki://Метод_Феррари
Найти такой простой формульный алгоритм для многочленов более высоких степеней невозможно, это доказано.

Но можно на компьютере находить корни многочленов более высоких степеней можно численными методами. Для этого надо сначала, найти отрезки, которым принадлежат корни, чтобы ни в одном не было более одного корня. Это можно в общем виде для многочленов сделать с помощью полиномов Штурма. Зная, что на данном отрезке есть ровно один корень его можно численно посчитать с любой наперёд заданной степенью точности каким-нибудь численным методом решения уравнений, например, методом Ньютона, методом дихотомии, методом простой итерации. Только там есть всякие оговорки на условия применимости, аккуратно.

Про рациональные функции. Так называют все функции вида многочлен на многочлен. Точно, что если степень числителя больше либо равна степени знаменателя, то можно делением числителя на знаменатель найти представление в виде многочлен + рациональная дробь, у которой числитель имеет строго меньшую степень чем знаменатель.
ruwiki://Деление_многочленов_столбиком

Дальше осторожно, линейная алгебра:
В принципе ясно, что многочлен это тупо последовательность коэффициентов перед х:
v = (a1,a2,a3,a4,...) - бесконечный вектор. То есть коэффициенты многочлена - это его координаты. А сам многочлен - это точка в бесконечномерном пространстве. Со всем вытекающим! На этой ноте мне пора прекращать монографию, понимая, что дальше при таком обобщении можно всю линейную алгебру переписывать, говоря, что её можно применять к многочленам, так как это векторы. Но я ещё напишу кое-что. В линалге говорят о линейных операторах. Это любые преобразования A(v), получающие из одного вектора другой, такие что:
A(v+w) = A(v) + A(w)
A(αv) = αA(v)
Это называется свойствами линейности. α - это число, на которое мы умножаем наш многочлен. v и w - многочлены, которые мы попросту складываем. Фишка в том, что если рассматривать только многочлены степени, не превышающей n, и А, которое, изменяя многочлены, тоже не сделает из них многочлены степени выше n, то воздействие A на многочлен можно свести к умножению на матрицу. Матрица оператора A отыскивается так:
Берём многочлены 1, x, x^2, x^3, x^4,... - то есть в координатах на одном месте единичка, а остальные - нули. Например, если рассматриваем многочлены не выше 4 степени, то 1, x, x^2, x^3, x^4 берём. Воздействуем на них А. Получаем сколько-то(n+1, 5 например, 5 же было) новых многочленов. У них тоже должны быть координаты, n+1 штука. Вот эти координаты записываем столбцами матрицы. Теперь умножая на эту матрицу, как на пикрелейтед, можно вычислять A(v).

А теперь примеры. Производная, вторая производная, интегралы от многочленов. Они линейны и для ограниченной степени можно найти их матрицы и легко вычислять потом интегралы и производные многочленов путём умножения на матрицу. Кстати для обычных трёх- и двумерных векторов линейны всякие изменения размеров, вращения и т.п. И их тоже можно так вычислять.

Фух. Всё написал, что можно было про многочлены или что-то осталось?
>> No.92669 Reply
>>92646
Опыт.
>> No.92670 Reply
>>92646
Там и ходов-то никаких нет. Неполный квадрат в знаменателе сразу бросается в глаза.
>> No.92674 Reply
>>92651
> А теперь примеры. Производная, вторая производная, интегралы от многочленов. Они линейны и для ограниченной степени можно найти их матрицы и легко вычислять потом интегралы и производные многочленов путём умножения на матрицу. Кстати для обычных трёх- и двумерных векторов линейны всякие изменения размеров, вращения и т.п. И их тоже можно так вычислять.
Можно пример?
>> No.92684 Reply
Котики, посоветуйте как освоить интегрирование, ну я вот реально не понимаю его, там всякие хитрые ходы надо делать по пути, потом всякие мелкие нюансы знать надо, ну вообще охренеть можно.
>> No.92687 Reply
File: 23022013043.jpg
Jpg, 378.19 KB, 2592×1944 - Click the image to expand
edit Find source with google Find source with iqdb
23022013043.jpg
>> No.92723 Reply
>>92651
Спасибо. Этого текста мне на долго хватит.
>> No.92725 Reply
>>92684
Увы, только задрачиванием примеров.
В качестве контроля можно использовать ответы в задачнике (могут быть опечатки) и maxima.
Сначала задрачивают стандартные ходы - замена переменной, затем интегрирование по частям.
Причем следует заранее послать нахуй быдлоучителей, которые объясняют интегрирование по частям формулой
У на дэвэ минус интеграл вэ на ду.
Потому как при этом происходит зацикливание неокрепшего фимозга студента на поиске этих самых у и вэ в формуле.
Суть формулы состоит в перекидывании производной с одного куска произведения на другой. Чтобы грамотно ее применять,
надо выучить ее вывод, основанный на определении производной произведения.

99% школоинтегралов беруться этими двумя штуками.
Дальше начинается упарывание всякими веселыми штуками типа интеграла от дроба.
Суть состоит разложении дроба на сумму простых дробов, тогда получим сумму легко берущихся интегралов.

В практике вычислительной математики чаще всего вылезает интегрирование по частям
(обычно части берут так, чтобы контурный интеграл обнулился, PROFIT!!11).

Упоротые интегралы с тригонометрией внутри обычно берутся maximой, во избежание разрыва мозга.
>> No.92726 Reply
>>90593
Котаны, кто-нибудь решал большие огромные разряженные системы линейных алгебраических уравнений?
GMRES с ILU предобуславиливанием и предварительной симметризацией работают у меня как-то не так.
ЧЯДНТ?
>> No.92728 Reply
Няши, посоветуйте учебник по дифурам, чтобы был написан с использованием топологии?
>> No.92729 Reply
File: 1338733747001.jpg
Jpg, 455.82 KB, 3679×2592 - Click the image to expand
edit Find source with google Find source with iqdb
1338733747001.jpg
>>92728
Няша, если ты не способен отличить повествовательное предложение от вопросительного, то тебе лучше математикой не заниматься. Сделай лучше что-нибудь приятное для людей. Двор подмети, например.
>> No.92730 Reply
>>92729
Анон, ты ведь понимаешь, что твои суждения о том, чем мне стоит заниматься, не интересуют меня? И понимаешь, что написание постов, имеющих целью лишь демонстрацию наличия у тебя огромного ЧСВ, никому не приносит пользы? Если не можешь посоветовать книжку, то лучше промолчи, пожалуйста, а если можешь, то сделай приятное и посоветуй. Доброчан же.
>> No.92737 Reply
>>92730
Но ведь на этом пути тебя ждут только унижения. Не ходи в математику, няшик, ты будешь путаться в простейших построениях.
>> No.92739 Reply
>>92737
Пичалька.
Книжку посоветуешь, не?
>> No.92774 Reply
А какую область математики можно изучать с нуля, не в классической математике?
>> No.92780 Reply
>>92774
В математике все связано, так что ты не взлетишь.
>> No.92785 Reply
>>92780
Я мечтал о такой игре, где все всё связано. А сейчас мечтаю о какой-то хуйне, — как быстро меняются мечты!
>> No.92802 Reply
File: limonov_001.jpg
Jpg, 102.41 KB, 500×303 - Click the image to expand
edit Find source with google Find source with iqdb
limonov_001.jpg
>>92726
Нахуя вы их называете разряженными, мудаки.
>> No.92806 Reply
>>92774
>>92780
Можно, например, посмотреть азы топологии. Они довольно чудные и не связаны с арифметикой. Еще есть теория вязания узлов, но я не помню, как она правильно называется, она тоже непонятная.
>> No.92822 Reply
Добрач, я не понимаю как доказывают через определения пределов через язык свой дебильный эпсилон дельта.
Вот дан предел при z-> 3-4i |z|=5

Ну вот допустим я начинаю писать при любом эпсилоне больше нуля существует такое дельта от эпсилона что ||z|-5<дельты а дальше то что?
>> No.92826 Reply
>>92822
Найди функцию delta(epsilon) = delta, где z in V0delta(3-4i) => |z| in V0epsilon(5).
>> No.92832 Reply
Кафедра, вопрос к вам из /b/. Есть десятичная дробь d конечной длины из отрезка [0;1].
Всегда ли можно подобрать два числа a и b таких, что
1)a/b = d
2)суммарное количество знаков в a и b меньше количества знаков в d?
>> No.92843 Reply
>>92832
Первое - всегда.
Второе - если a и b не взаимно просты (дроб сокращаем).
Так как b у нас дано свыше (дроб конечный и десятичный), следовательно, начальное a тоже дано свыше.
И вот если это a поделится на b, тогда таки да, сокращение произойдет.

Так шта тот самый школолольник эпично сосет. Как обычно впрочем.
>> No.92844 Reply
>>92832
нет
0.9 2 знака
9/10 3 знака
18/20 > 3 знаков
>> No.92845 Reply
>>92843
Ну тут стоит учесть, что один общий делитель у них есть, и он дает d.
А вот если у них есть другие общие делители, тогда можно и сократить.
>> No.92846 Reply
>>92843
Тут еще можно пофантазировать на тему цепных дробей, как самого экономного способа записи обыкновенных дробей вообще.
>> No.92848 Reply
>>92843
Если кто не в теме, школьник предлагает способ архивирования файлов. Суть: файл - это большое двоичное число. Переводим его в десятичную систему, записываем впереди 0, получаем десятичную дробь из [0;1). Затем подбираем два числа a и b такие, что их частное даст нашу десятичную дробь.

Школьник предполагает, что это позволит упаковывать многогигабайтные файлы в сравнительно небольшие архивы. Архивирование файла может занять много времени. Разархивирование производится делением a на b весьма быстро.
>> No.92853 Reply
>>92848
На сжатие есть природное ограничение в виде Колмогоровской сложности. Посоветуйте товарищу почитать щито это, раз уж он интересуется такими вещами.
>> No.92854 Reply
File: 02495889.jpg
Jpg, 99.28 KB, 800×600 - Click the image to expand
edit Find source with google Find source with iqdb
02495889.jpg
>>92728
Совет очень нужен, няши.
>> No.92855 Reply
>>92854
Рохлин - начинал в этой стране с этим направлениемв. Искать среди печатных работ СПБГУ.
>> No.92882 Reply
>>92854
Арнольд, "Обыкновенные дифференциальные уравнения", разумеется
Если в одном предложении встречаются слова "дифуры" и "топология", то подумай о книжках Арнольда в первую очередь.
>> No.92903 Reply
Аноны, для чего используют мапл? Он по ходу для быстрых вычислений с большими объёмами входных данных вообще не пригоден, что ль? Какой-то он непредсказуемый. Умножение квадратной матрицы на вектор даёт почему-то вообще массив, а не вектор и выписывает его в строкой через запятую при выводе. Хотя для больших входных данных такой вывод не будет юзаться, всё же странно...
>> No.92918 Reply
>>92903
Мапл пригоден для небольших СИМВОЛЬНЫХ вычислений.
Если ты хочешь цифродробилку для матриц, бери BLAS (Хотя оно говно еще то, написанное на фортране 30 лет назад,
с упоротыми сигнатурами в венгерской нотации и кучей лишних параметров).
Есть еще GSL, оно уже на Си, но BLAS в нем просто портирован, так что все равно говно.
А некоторые кококок - швабодка библиотеки тупо содержат баги, влияющие на точность.
То есть для маленьких примеров все шоколадно, а как хуйнешь ей матрицу 10000000x10000000, так пиздец наступает, хотя MKL такую матрицу щелкает.
но MKL-не швабодка, у нее ональная лицнзия.

Хочешь скорости - пейши на Си, Поцкале или Фортране (но последний упорот и в нем приходится чаще ебаться с неочевидностями в стандарте языка, чем с собственными багами). Поц плох тем, что в нем нет нативной поддержки OMP, а распараллеливание нахаляву порой так нужно.

Вывод - Хочешь цифродробилку - пейши свое на Си.

А баги вот такие, например - при сложении кучи чисел с плавающей точкой, модули которых существенно разнятся,
результаты сложения отсортированного по убыванию набора и не сортированного существенно различаются. И при этом результат для сортированного точнее

И такого говна в вычислительной математике километры.
>> No.92943 Reply
File: 1361888124501.png
Png, 0.83 KB, 300×20 - Click the image to expand
edit Find source with google Find source with iqdb
1361888124501.png
>>92848
> Если кто не в теме, школьник предлагает способ архивирования файлов. Суть: файл - это большое двоичное число. Переводим его в десятичную систему, записываем впереди 0, получаем десятичную дробь из [0;1). Затем подбираем два числа a и b такие, что их частное даст нашу десятичную дробь.
Неплохо придумано.
>> No.92947 Reply
>>92943
Теорему Котельникова-Шеннона опроверг? Ну-ну.
>> No.92983 Reply
>>92854
Свадьбу играй, няша!
>> No.92990 Reply
>>92983
Не свадьбу, а БРАК.
>> No.93024 Reply
Джентльмены, будьте добры, подскажите какую-нибудь статью про задачу о клике, варианты алгоритма нахождения, алгоритм Брона-Кербоша etc. Если такого нет, то какую-нибудь обзорную статью по NP-полным задачам, желательно, опять-таки, с алгоритмами.
>> No.93028 Reply
>>92990
Хорошее дело браком не назовут. Вангую что слово брак какие нибудь засранцы начали использовать для обозначения этого дела первыми
>> No.93029 Reply
>>93024
Ну сколько можно этот баян мусолить?!
>> No.93032 Reply
>>93029
Серьезно? Уже было? Прошу прощения. Сижу в математических тредах достаточно давно, даже создал один, но дискретная математика никогда не попадала в область моих интересов, так что все посты, связанные с ней, проходили мимо моего сознания, а сейчас стало нужно. Может, все-таки, у кого-нибудь есть что-то подобное?
>> No.93033 Reply
>>93032
Это я к тому, что большинство быдлокодеров, заслышав про P?NP тут же хватаются за какой - нибудь баян типа коммивояжора или поиска клики.
Ты из таких будешь?
>> No.93034 Reply
>>93033
Не, я матшкольник с сильно разбросанными математическими интересами. Олимпиаду по программированию пишу, выяснилось, что одна задача сводится к поиску клики, пытаюсь теперь найти что-нибудь по этой теме.
>> No.93035 Reply
>>93034
Классическим для современного интернетчика стилем является старт с педивикии по ссылкам.
Так как олимпиадоту судят учителя и доценты за еду, рикамендую особо не выебываться, а хуйнуть баян системы "метод ветвей и границ" (это как раз вооон та хуйня, обозванная ЕРЖ фамилией (ну куда без них-то). В противном случае есть риск получить хуй-с за малоизвестное решение - точно решат, что спиздил.
С разбросом математических (да и любых) интереснов надо люто, бешено бороться (это как встречаться с тремя-пятью-кто-больше бабами зараз), иначе это приведет в дальнейшем к пиздецу - нормальные отношения построить не удастся ни с одной, а драгоценное время и ресурсы головы будут просраны.

Так и здесь, вместо убийства времени на поиск заебатого решения,
лучше хуйнуть стандарт и чесать дальше, а не выдрачивать одну задачу до бесконечности.

Я вот в свое время тоже в диссертации дрочил-дрочил алгоритм, получил выигрыш 3-5%, а убил пол-месяца. Вот нахуя оно?
>> No.93039 Reply
>>93035
> С разбросом математических (да и любых) интереснов надо люто, бешено бороться
И в то же время
> вместо убийства времени на поиск заебатого решения, лучше хуйнуть стандарт и чесать дальше, а не выдрачивать одну задачу до бесконечности
>> No.93040 Reply
>>93039
Какие-то проблемы? У вас квадратногнездовье мозга? Вы таки не понимаете, что неоднозначные интересы суть есть попытка скакать с топологии на диффуры с диффур га ГА, с ГА на линал, с линала еще дальше?
>> No.93041 Reply
>>93040
А почему вы называете это скачкой? Это азы, которыми должен владеть любой математик. Скачка - это, например, попытка одновременно углубиться и в теорию колец, и в теорию кос и узлов.
>> No.93042 Reply
>>93035
> С разбросом математических (да и любых) интереснов надо люто, бешено бороться
лол, тупее совета и не придумать
"математик", который задрачивал только одну область, к науке не способен и никому в научном плане не интересен, невежд итак очень много
>> No.93043 Reply
>>93041
Я об этом и говорю. Знать о нужно все разделы, но углубляться - в один и смежные, а не во все сразу.
>> No.93044 Reply
>>93041
> углубиться и в теорию колец, и в теорию кос и узлов
это тоже неудачный пример, хотя что ещё называть "углубиться", любой из знакомых мне практикующих математиков довольно неплохо знает и кольца и топологию малых размерностей (т.е. узлы), просто это вещи очень базовые.
>> No.93045 Reply
>>91700
1995 год -- доказана Великая Теорема Ферма, но я такой слоу, что узнал про это только позавчера
>> No.93046 Reply
>>93045
Ну гипотеза Пуанкаре же!
>> No.93047 Reply
File: 1205442042834.jpg
Jpg, 78.37 KB, 1018×739
Your censorship settings forbid this file.
r-18
Аноним/u/с, подкинь чего-нибудь по решению неопределенных уравнений. Диофант, конечно, наше все, но уже несколько устарел. Посоветуйте чего-нибудь свеженького.
>> No.93048 Reply
>>93046
уже упоминали в треде
>> No.93049 Reply
>>93035
> С разбросом математических (да и любых) интереснов надо люто, бешено бороться (это как встречаться с тремя-пятью-кто-больше бабами зараз), иначе это приведет в дальнейшем к пиздецу - нормальные отношения построить не удастся ни с одной, а драгоценное время и ресурсы головы будут просраны.
Какой однобокий подход.
>> No.93050 Reply
>>93040
Да, Пуанкаре смотрит на тебя как на флюс.
>> No.93051 Reply
File: 1003272093.jpg
Jpg, 224.68 KB, 473×700 - Click the image to expand
edit Find source with google Find source with iqdb
1003272093.jpg
>>93044
> хотя что ещё называть "углубиться"
http://yadi.sk/d/3kmX1V6m2xVjN

Например.
>> No.93052 Reply
>>93045
>>93046
А ещё что?
>> No.93062 Reply
>>93033
Это важные теоретически знания. Но смысл. решать эти задачи на практике я не знаю.
>>93034
Эти "байаны" - NP-полные задачи. Все задачи класса NP сводятся к любой NP-полной. Любую NP-задачу можно решить за экспоненциальное время. Решить их быстрее чем за полинамиальное в общем случае невозможно(То есть точно не все.) Можно ли быстрее экспоненты, на пример собственно за полином? Вот над эти вопросом и бьются. Если бы NP-полную решили бы быстрее чем за экспоненту, то за такое время решились бы все задачи класса NP. Сказав, что задача сводится к клике тебе наверное решили намекнуть, что быстрее экспоненты ты её не решишь. Хотя не факт. Это если она сама была NP-полной, например, если другая NP-полная к ней сводилась. Кстати, все задачи, решаемые за полиномиальное время(класс P) относятся к NP.
>>93042
Тем не менее никто не знает одинаково хорошо все области.
>> No.93070 Reply
>>93062
> Кстати, все задачи, решаемые за полиномиальное время(класс P) относятся к NP.
> Если бы NP-полную решили бы быстрее чем за экспоненту, то за такое время решились бы все задачи класса NP.
Хм.
>> No.93075 Reply
>>93070
NP ещё не значит NP-полная
>> No.93076 Reply
>>93070
Эм, если я написал, что я школьник, то это значит, что я идиот и не знаю этого? И мне никто ничего не говорил, я свел ее к клике самостоятельно. Спасибо большое, растолковывай очевидности кому-нибудь другому. Реквест тот же, подборка алгоритмов, хотя он уже менее актуален, я решил задачу методом ветвей и границ.
>> No.93080 Reply
>>93076 Не туда, прошу прощения, этот пост предназначался >>93062
А я его еще и прочитал невнимательно... Ты же ведь ерунду написал. И еще кому-то ее всучивать пытаешься.
>> No.93081 Reply
>>93080
Поясни мне, где конкретно я написал "ерунду". Я хотел подвести к тому, что клику по любому можно решить за экспоненту(основание может быть и не e, а другим), а если решишь быстрее или докажешь, что нельзя решить быстрее, то медаль Филдса получишь. Если докажешь, что нельзя за полином или решишь за полином, то это решит проблему тысячелетия.
>> No.93083 Reply
>>93081
P.S., под сведением здесь понимается сведение за полиномиальное время. То есть решение за полином операций.
> Если бы NP-полную решили бы быстрее чем за экспоненту, то за такое время решились бы все задачи класса NP.
Не за такое время, а за полином от такого времени.
>> No.93107 Reply
>>93090
Что, для тебя одной стараться?
>> No.93108 Reply
>>93083
Да, тогда правильно, извини. Не совсем понял, что понималось под сведением.
>> No.93115 Reply
>>93107
Что ты хочешь сказать? Я же прошу всего пару занятий со мной провести и даже готова заплатить за это.
>> No.93116 Reply
>>93115
> и даже готова заплатить
Да что ты можешь дать, девочка? Пять-десять тысяч деревянных? Оставь себе свои копейки.
> Что ты хочешь сказать?
Что публичная беседа на Доброчане лучше приватного диалога в скайпе - она принесёт пользу не только тебе. Скажи, про что хочешь послушать и что хочешь уметь, и я расскажу про это здесь. Только предварительно опиши, что тебе уже известно, и скажи, зачем тебе всё это надо, - контрольную сдать или саморазвитием пострадать.
>> No.93122 Reply
Есть ли здесь первокурсницы, желающие изучить общий курс высшей математики по скайпикув кафе на квартире за деньги безвозмездно? (Конкретно буду рассказывать топологию? гомологическую алгебру и какую-нибудь геометрию по выбору)
>> No.93125 Reply
>>93116
> Что публичная беседа на Доброчане лучше приватного диалога в скайпе - она принесёт пользу не только тебе.
Лол. Девочка, не слушай этого дядю.
>> No.93132 Reply
>>93122
И этого слушать не надо. Тоже мне, Льюис Кэролл.
>> No.93133 Reply
>>93125
>>93132
Жадный студент, залогинься.
>> No.93134 Reply
>>93133
Я не жадный студент, я добрый отец, который вас, педобирей спермотоксикозных, за 9000 км чует.
>> No.93135 Reply
File: 23628.jpg
Jpg, 107.54 KB, 800×600 - Click the image to expand
edit Find source with google Find source with iqdb
23628.jpg
>>93134
> первокурсница
> педобирей
>> No.93136 Reply
>>93135
17 лет, попизди мне тут
>> No.93137 Reply
>>93136
В РФ возраст согласия 16 лет, есличо

И вообще, ты какой-то грубиян, пойди-ка нахуй
>> No.93138 Reply
>>93122
Есть просто кун. Пойдет?
>> No.93140 Reply
>>93137
При условии замужества, есличо.
>> No.93141 Reply
>>93140
Пруф или пшёл прочь с маткафедры.
>> No.93142 Reply
>>93141
Я фейл, ошибся с возрастом в статье 135 п.1 УК РФ.
>> No.93143 Reply
>>93140
Слушай, ну, тебе было бы разумнее не продолжать дискуссию, раз уж ты такой тупой.
Возраст согласия обозначен в УК (статья 134)
Брачный возраст по дефолту с восемнадцати ("Семейный кодекс"), хотя в некоторых областях меньше.
>> No.93144 Reply
>>93143
Высказался и аж потеплело, да?
>> No.93145 Reply
File: cute.jpg
Jpg, 34.91 KB, 600×450 - Click the image to expand
edit Find source with google Find source with iqdb
cute.jpg
ТНИ! НЕ! НУЖНЫ!
>> No.93146 Reply
>>93145
ЛЮДИ! ЛЮДИ! НЕ! НУЖНЫ!
>> No.93149 Reply
Анон, я за консультацией. Я учил матан в универе, но учил не тем местом. Итог: я знаю основыные курсы, но крайне поверхностно. Как мне стать гуру математики ну или просто выучить её нормально? Нужно ли начать с нуля для этого?
>>93122
Я желаю. Но я четверокурсник, это не страшно?
>> No.93151 Reply
>>93149
> Нужно ли начать с нуля для этого?
Прочитай учебник Зорича. Он простой и понятный. Пригодится.
>> No.93153 Reply
File: coffin_for_obese.png
Png, 236.17 KB, 516×292 - Click the image to expand
edit Find source with google Find source with iqdb
coffin_for_obese.png
>>93133
Ну кокой же я жадный студент, просто тут, похоже, имеет место неверное понимание сути матемача - он нужен для обсуждения Арнольда и толстоты про периодическую девятку. По крайней мере - точно не для помощи всяким девочкам.
>> No.93157 Reply
File: 1362223158025.png
Png, 0.91 KB, 300×20 - Click the image to expand
edit Find source with google Find source with iqdb
1362223158025.png
>>93153
Может быть ей уже помогли в скайпике.
Что можно написать в посте на имиджборде такого, что уже не написано в учебнике или в лекциях? Тут даже формулы нельзя вставить.
>> No.93158 Reply
>>93157
У учебников и лекций нет обратной связи.
>> No.93159 Reply
File: x_319e9900.jpg
Jpg, 49.79 KB, 307×604 - Click the image to expand
edit Find source with google Find source with iqdb
x_319e9900.jpg
>>93138
> просто кун. Пойдет?
>> No.93160 Reply
>>93159
Фрик такой фрик...
В школе геем травили? Боишься с мужчинами заговорить? Боишься, что просекут о твоих КОРИЧНЕВЫХ намерениях?
>> No.93163 Reply
>>93153
А при чём тут Арнольд? Зачем вы его впутываете?
>> No.93165 Reply
>>93163
Ну это же главный популяризатор математики в этой стране
>> No.93166 Reply
>>93165
Лолшто? Арнольд, при всех его заслугах, главный популяризатор Арнольда.
>> No.93167 Reply
>>93166
Он популяризатор Арнольда в математике
>> No.93173 Reply
>>93167
Не только в математике.
lm://Принцип_Арнольда
>> No.93174 Reply
File: 034987.jpg
Jpg, 88.37 KB, 512×469 - Click the image to expand
edit Find source with google Find source with iqdb
034987.jpg
>>93167
А что Арнольд ещё и в математику может?!
>> No.93176 Reply
>>93174
А теперь пошути про матрицы, линейную алгебру и Нео.
>> No.93177 Reply
>>93176
Вот ему чаю дайте!
>> No.93189 Reply
>>93173
Ничего себе! Целую статью накатали.
> много и часто, даже иногда и по делу, ругал и критиковал мехмат
> даже иногда и по делу
а-ха-ха-ха
>> No.93191 Reply
>>93189
Все-таки, мехмат это совсем плохо, даа?
>> No.93193 Reply
>>93191
Ситуация такая.
В 60-е мехмат МГУ был одним из сильнейших математических факультетов в мире, может, даже самый сильный.
В 70-е дело пошло под откос, в 80-е наступил пиздец, а в конце 80-х, когда открылись границы, и корифеи науки, да и вообще почти все мало-мальски образованные люди с мехмата решительно ломанулись (причём не из-за финансовых проблем, тому же Арнольду, чтобы жить безбедно, достаточно было совершать пару месячных поездок в год). Тогда мехмат и превратился в гнилую парашу, в коем состоянии он и пребывает поныне.
>> No.93194 Reply
А причиной скатывания в говно было то, что власть на мехмате и во всём мгу постепенно захватывали люди, продвигавшиеся "по партийной линии", т.е. к науке не имевшие отношения. Разумеется, такие управленцы старались изжить настоящих учёных всеми способами.
>> No.93200 Reply
>>93193
Доказательства есть?
>> No.93202 Reply
>>93200
А что именно ты хочешь, чтобы я тебе доказал, мой маленький друг?
>> No.93203 Reply
>>93202
> мехмат и превратился в гнилую парашу, в коем состоянии он и пребывает поныне
this
>> No.93205 Reply
>>93203
Расписания, учебные планы и списки спецкурсов в открытом доступе.
"Имеющий уши, да услышит", ну а в данном случае увидит.

А увидит он вот что: то, чему можно научиться на мехмате, не актуально уже по состоянию на 30-е годы. А уж то, что происходило после 40-х (т.е. вообще всё самое главное в математике), не представлено и вовсе, подчистую. Кроме того, очень низкий,за редчайшим исключением, преподавательский состав: недоучки остаются у себя на кафедре, и невежество воспроизводит невежество.

В итоге имеем то, что имеем. Выпускник мехмата, не ходивший в НМУ или ещё куда-то в этом роде, может стать программистом, менеджером, бухгалтером, но уже не может стать математиком.
>> No.93209 Reply
>>93205
> Расписания, учебные планы и списки спецкурсов в открытом доступе. "Имеющий уши, да услышит", ну а в данном случае увидит.
"Критикуя, критикуй". Что не так с курсами и расписанием?
> то, чему можно научиться на мехмате, не актуально уже по состоянию на 30-е годы. А уж то, что происходило после 40-х, не представлено и вовсе, подчистую.
Как это сочетается с утверждением "в 60-е мехмат МГУ был одним из сильнейших математических факультетов в мире, может, даже самый сильный"?
> т.е. вообще всё самое главное в математике
Например что?
>> No.93211 Reply
>>93209
> Что не так с курсами и расписанием?
То, что они устарели на 80 лет и вместо образования калечат.
> Как это сочетается с утверждением
Это современное образование на мехмате абсолютно бессмысленно и отстало.
В шестидесятые-то на мехмате работали выдающиеся люди, у них были семинары, о которых сейчас легенды ходят, на этих семинарах можно было учиться науке.
> > т.е. вообще всё самое главное в математике
> Например что?
В период сороковых-шестидесятых были созданы алгебраическая геометрия, алгебраическая топология, гомологическая алгебра, К-теория, существенно углублена теория групп Ли, дифференциальная геометрия, дифференциальная топология. Очень много чего ещё, конечно.

Это был золотой век, лучшее что случалось в математике когда-либо, и вряд ли повториться в ближайшее время. Никогда прежде не было такого плодотворного периода, когда открывались целые миры. То, что происходило в те времена, важно для математики так же, как для физика важны открытие электромагнетизма, квантовых эффектов и общей теории относительности одновременно.

Потом был "серебрянный век" 70-80 с прорывами в теории представлений и развитием мощнейшей гомологической техники, а ещё комплексная геометрия, да много чего ещё.

Так вот, выпускник мехмата абсолютно ничего этого знать не будет.
>> No.93213 Reply
>>93211
Позвольте, но по вашим словам выходит, что все великие события в математике отгремели, и сейчас нет открытий, которые могли бы по значимости сравниться с открытиями шестидесятых. Неужели это в самом деле так?
>> No.93214 Reply
>>93213
Да, сейчас концептуальное развитие математики замедлено, некоторые даже называют это застоем.
Можно сказать, что в 50-е были обнаружены богатейшие месторождения, о которых люди раньше и не догадывались, а сейчас из этих месторождений выгребают последние остатки, доступные при имеющейся технике.

Но науке известно о математических объектах очень и очень мало, так что нет сомнений, что будут и новые концептуальные прорывы.
>> No.93216 Reply
>>93213
Ну и, кстати, из моих слов-то это не выходит, но вы правильно угадали.
>> No.93217 Reply
>>93216
Ну, золотой век был, серебряный век был, стало быть, сейчас не золотой и не серебряный век. Так и выходит.
А с чем была связана эта революция? Что привело к такому взрыву открытий?
>> No.93218 Reply
>>93217
Очевидно, что вторая мировая, а потом и холодная война. Мощнейшие вливания в инженегров, которые так или иначе докатились до теоретиков.
Я должен заметить, что пиздец в математике везде, а не только в МФТИ. Ровно как и пиздец в информатике, физике, etc.
Если уже в университетах выпускают методички с фактическими ошибками.
>> No.93219 Reply
>>93217
> А с чем была связана эта революция?
Сложный вопрос.

В одно время и в одной науке собрались люди чрезвычайной интеллектуальной силы, а некоторые личности просто какого-то мистического уровня.
К тому же реализация творческой энергии, накопленной за время войны.
Ну и, конечно, плечи гигантов: Пуанкаре, Гильберт, Нётер, Риман и другие.
Да и в конце концов когда-нибудь должен был произойти этот прорыв на следующий уровень, вот он и произошёл.
>> No.93231 Reply
File: 1354.png
Png, 42.27 KB, 486×604 - Click the image to expand
edit Find source with google Find source with iqdb
1354.png
>>93116
Мне нужно подготовиться к контрольной и сделать домашнее задание.
Но я совсем не понимаю алгоритма решения этих заданий. Общие правила и действия мне известны, но с конкретными заданиями не выходит.

Буду рада хотя бы парочке решений или алгоритмов решения.
>> No.93232 Reply
>>93231
Учу матан по этой книжке. Все алгоритмы есть. Стр. 226-256. http://rghost.ru/44231376
крокодил
>> No.93234 Reply
Доброаноны, привет. У меня есть пара недель, чтобы как следует разобраться в таких топиках:
> Метрические пространства.
> Компактность.
> Нормированные и евклидовы пространства.
> Теория меры и интеграла Лебега.
> Ограниченные операторы в банаховых и гильбертовом пространствах
Какие будут предложения?
>> No.93237 Reply
>>93234
Рудин
Колмогоров, Фомин
>> No.93239 Reply
>>93231
мимокрокожу: кому нафиг сдалось это закорючкоразгибательство? Все равно к 3-4 курсу не-математики(включая инженегров) забывают все сложнее логарифмов, ибо не нужно оно в работе. А интегралы численно железяка разогнет. Или символьно - умная железяка.
>> No.93242 Reply
>>93231
Вот это (на пикрелейтед) чистейший экстракт соков говна, просто эталон.
>> No.93243 Reply
А потом всякие няши будут думать, что вот это и есть математика.
>> No.93248 Reply
>>93242
раздался голос со стороны отчисленной первокурсоты
>> No.93250 Reply
File: котэ-дельфин-гифки-животные-80744.gif
Gif, 14396.38 KB, 309×219 - Click the image to expand
edit Find source with google Find source with iqdb
котэ-дельфин-гифки-животные-80744.gif
>>93237
Ага, спасибки, так и планировал. Единственное - мне почему-то нирикамендовали Колмогорова-Фомина.
>>93248
Раздался голос залетного сосачева
>> No.93251 Reply
File: 0412da70494411e2a63622000a9e28ec_7.jpg
Jpg, 91.49 KB, 612×612 - Click the image to expand
edit Find source with google Find source with iqdb
0412da70494411e2a63622000a9e28ec_7.jpg
>>93250
> котэ-дельфин-гифки-животные-80744.gif
Блядь, стыдно-то как.
>> No.93257 Reply
>>93248
Не, это няша. Он мне книжки посоветовал, благодаря которым я вывел смысл жизни.
>> No.93258 Reply
>>93250
> Раздался голос залетного сосачева
Так он же прав, этот кун был отчислен ещё когда ему было 17-19, т.е. относительно недавно.
Это если я его не спутал с другим неймфагом.
>> No.93259 Reply
>>93159
Сру тебе в рот. Думать о поле собеседника во время обучения математике - неподобающая мерзость. Не мешай математику своими сексуальными фантазиями. Фу, говорить с такими.
>> No.93260 Reply
>>93258
Извините, у меня как у человека, набравшегося в свое время интегралов, на любого их защитника такая реакция.
>> No.93261 Reply
>>93259
Учить няшных девочек приятнее, чем учить кунов, потому что кунов приходится учить на самом деле.
>> No.93263 Reply
>>93260
А что тут набираться? Примеры выше тривиальны. Научиться "брать интегралы" в первокурсном смысле можно за один вечер.
> набравшегося в свое время интегралов
Почему вы так не любите интегралы? Интеграл - полезнейшая и важная вещь в математике.
>> No.93264 Reply
>>93263
Вернулись бы вы, в самом деле, обратно.
>> No.93266 Reply
File: 1362315760388.png
Png, 1.49 KB, 300×20 - Click the image to expand
edit Find source with google Find source with iqdb
1362315760388.png
>>93264
К чему вы это?
>> No.93267 Reply
>>93263
Исчислять интегралы же. А те интегралы кои без спросу брал - возверни взад туда откель брал!
>> No.93270 Reply
>>93267
Нет, не исчислять. Это как раз-таки символьное интегрирование, а не численное. Я настаиваю на термине "брать".

ruwiki://Символьное_интегрирование
ruwiki://Численное_интегрирование
>> No.93272 Reply
>>93258
Ты не охуел, болезный?
Откуда это я был отчислен?


>>93263
> полезнейшая и важная вещь в математике
Это в какой математике-то символьное интегрирование является важной вещью? В той, что преподаётся в твой путяге? Это да, конечно.
Перед тем, как ответить мне что-нибудь остроумное, почитай предыдущие треды, этот вопрос уже не раз обсуждался.
>> No.93273 Reply
>>93272
> Это в какой математике-то символьное интегрирование является важной вещью?
Математика одна.
> В той, что преподаётся в твой путяге?
А ещё злее можешь, неймфаг? Либо ты немедленно объясняешь, почему символьное интегрирование - это зло, либо с треском сливаешься.
>> No.93275 Reply
>>93273
> Либо ты немедленно
Вот это новости! Оно мне будет командовать, лол.

Я всё понимаю, тебя так учили, ты жрал говно, думая, что это полезно, а теперь защищаешь эту точку зрения.
Ты такой не один, каждый, кто из математики видел только курс анализа в мехматовском стиле, имеет эту контузия, вас очень много.
Но разжёвывать одни и те же вещи много раз подряд мне просто лень. Полистай предыдущие треды, если будут вопросы -- обращайся.
>> No.93277 Reply
>>93275
Слив засчитан. Больше не вякай об интегралах.
>> No.93278 Reply
>>93273
Потому что это малополезное занятие. С одной стороны эта задача алгоритмически решена в общем виде, поэтому работа математиков здесь окончена, с другой стороны для инженеров такие задачи, вероятно, могут встречаться, но общее решение эффективно реализуется, и в связи с повсеместным распространением компьютеров нет никакого смысла мучить людей, когда за них это сделает компьютер гораздо быстрее.
Вообще говоря, математику вычислять интегралы не нужно, потому что к этому сводится никакая из современных математических проблем.
Можно иметь вполне хорошее хобби по вычислению интегралов - оно ничем не хуже, чем умножение 10-значных чисел в уме. Аутисты получают от этого удовольствие, лол.
>> No.93280 Reply
>>93278
Нашёлся добрый человек, разжевал.
> чем умножение 10-значных чисел в уме
Или ручной подсчёт определителей 10 на 10 -- тоже занятие ничем не хуже символьного интегрирования.
>> No.93281 Reply
Кстати, если спросить у говноеда, полезно ли уметь вручную считать определители 10х10, он же, наверное, скажет, что да.

Потому что те, кого в детстве пиздили, как раз и есть самые рьяные сторонники "воспитания" детей с помощью побоев и порок.

А те, кого ничему кроме интегралов не учили, есть самые рьяные защитники водофки и картофана.
>> No.93282 Reply
File: 153e1b069760443ba31b54c45ea85469.png
Png, 4.05 KB, 670×85 - Click the image to expand
edit Find source with google Find source with iqdb
153e1b069760443ba31b54c45ea85469.png
>>93278
Одни и те же аргументы, одни и те же лишённые смысла фразы. Я не понимаю, почему вы все так цепляетесь именно к интегралам? Мода, что ли? С тем же набором аргументов можно обрушиться вообще на что угодно. На производную, например, или на дифференциал, или на определение предела по Коши, или на ряды... Что там ещё на первом курсе проходят?
> С одной стороны эта задача алгоритмически решена в общем виде, поэтому работа математиков здесь окончена
А вы хотя бы в курсе, как именно она решена? Можете изложить вывод алгоритма Риша с полным обоснованием? Вы знаете, что сейчас нет ни одной компьютерной системы, которая бы могла более-менее вменяемо предоставлять пользователю возможность использовать этот алгоритм? Вот давайте посмотрим на пример, который есть даже в википедии: интеграл от x/sqrt(x^4 + 10x^2 - 96x - 71). Группа Галуа многочлена x^4 + 10x^2 - 96x - 71 D(4), поэтому интеграл легко берётся в элементарных функциях, а если поставить в знаменатель многочлен x^4 + 10x^2 - 96x - 72 (группа Галуа S(4)), то применение версии алгоритма Риша, которой пользуются железяки, не даст нам результата в виде композиции элементарных функций. Машины не чувствуют разницы и хреначат ответы в общем виде. Первообразная википедийного интеграла - пикрелейтед. Но полюбуйтесь на ответ вольфрам альфы! http://www.wolframalpha.com/input/?i=integrate+%28x%2Fsqrt%28x%5E4+%2B[...]29%29 Не правда ли, просто и интуитивно понятно? :-)
> с другой стороны для инженеров такие задачи, вероятно, могут встречаться
То есть как это - "вероятно"? Интегралы - это, помимо прочего, язык физики. Физику жизненно необходимо знать, как одни функции связаны с другими, для этого он и получает образование. Зачем обучаются символьному интегрированию представители других специальностей, можете посмотреть в учебных планах.
Вы, безусловно, можете требовать анафемствования интегралов. Вы можете забыть о том, что математика связана с реальным миром и уйти в "чистую математику". Вы сами выбираете, чем забивать свою голову, это ваше право. Но не пытайтесь навязать всем взамен математики вашу кастрированную химеру "чистой математики", она нежизнеспособна и почти никому не интересна. Во все времена мистиков и метатеоретиков отовсюду гнали. Недаром вы кичитесь своей малочисленностью. "Чистой математики" нет места в жизни. Она - урод.
> и в связи с повсеместным распространением компьютеров нет никакого смысла мучить людей, когда за них это сделает компьютер гораздо быстрее.
А также людям не нужно знать таблицу умножения - у них есть калькуляторы. И не нужно знать алфавит - зачем, ведь его всегда можно нагуглить. И не нужно уметь читать - ведь есть аудиокниги... Люди пришли учиться, они хотят иметь образование, причём не чисто-математическое, а такое, которое бы было полезно им в их деятельности. И потому им нужны интегралы. Тем более что они простые. Очень. Их не может понять только окончательно обленившийся человек.
> Вообще говоря, математику вычислять интегралы не нужно, потому что к этому сводится никакая из современных математических проблем.
Математик, не умеющий считать, не владеет математикой. Он юрист, лингвист, кто угодно - но не математик.
>> No.93284 Reply
>>93282
Чёрт. Опечатки.
>> No.93285 Reply
>>93272
> Ты не охуел...
Под Мисаку не ты косплеил?
>> No.93286 Reply
File: vodofka.jpg
Jpg, 100.13 KB, 720×482 - Click the image to expand
edit Find source with google Find source with iqdb
vodofka.jpg
>>93282
Гражданин графоман, с вами уже всё ясно, не напрягайтесь.
Ваш текст состоит на 45% из провинциальных штампов, на 45% из вашего батхёрта и на 10% из википедийной статьи.
Если вы не сумели в топологию, это не повод так злиться.

Лучше проходите к столу, я вам покушать принёс и выпить.
>> No.93287 Reply
>>93272
Да по тебе видно же, что ты странный, залётный какой-то.
>> No.93289 Reply
File: i-robot05.jpg
Jpg, 22.13 KB, 650×357 - Click the image to expand
edit Find source with google Find source with iqdb
i-robot05.jpg
>>93286
Товарищ неймфаг, вы всех, кто вам неприятен, считаете не умеющими в топологию батхертящими провинциальными злыднями, или же я вызвал у вас особенную неприязнь? Почему вместо конструктивной дискуссии вы скатываетесь в ругань и картофанчик с водовкой?
>> No.93291 Reply
>>93289
Если человек в дискуссии о математике начинает вспоминать про "ррреальную жызнь", это в 100% случаев оправдание невежества. Эмпирический факт такой.

Причём, вы, может быть, искренне так считаете, потому что вас так научили, или потому что вы ничего больше не видели.
Так вот, любая рутина, которая может быть автоматизирована, в том числе брание интегралов ручками или умножение 10-значных чисел, к математике отношения никакого не имеет. Если вас заставляли это делать, то знайте: вас наебали. Тут пользы столько же, как если стакан соли сожрать.

>>93285
нет, не я
>> No.93292 Reply
File: irobot147464160426.jpg
Jpg, 279.74 KB, 1280×544 - Click the image to expand
edit Find source with google Find source with iqdb
irobot147464160426.jpg
>>93291
Дам вам факт взамен: если человек направо и налево сыпет процентами, то он инфантилен.
Мы говорим об образовании, а не о чистоте математики (tm). Образование получают для того, чтобы работать. Машина, которая вместо простого и понятного ответа выдаёт ахинею - слабое подспорье в работе. Человек, который зависит от машины даже в азах, - не инженер, а кнопкодав. Здесь можно провести аналогию со владением языком: один человек говорит свободно, другой через слово лезет в словарь. Понятно, какой человек имеет образование, а какой нет.
> Так вот, любая рутина, которая может быть автоматизирована
Но зачем до абсурда-то доводить? Всё хорошо в меру, и автоматизация в том числе. Поливать грязью интегралы и требовать полной отмены их изучения даже профилями - глупость. Так можно прийти к заявлению, что ничему учиться не надо, потому что всё делают машины.
А ещё вы маскируете желание поотменять к чертям собачьим всё самое важное необходимостью обновить курс подготовки инженеров.
> или потому что вы ничего больше не видели
Вы сами-то многое видели? Очень похоже на то, что вы бездумно повторяете слова какого-то вашего авторитета и матом-петросянством-самовозвеличиванием пытаетесь не дать себе усомниться в нём.
>> No.93293 Reply
>>93292
Уметь брать интегралы от синуса, косинуса, многочленов и ещё чего-то в этом духе нужно. Просто потому, что это очевидно. Так же, как знать таблицу умножения, её как бы трудно не знать. А вот специально изучать разные приёмы символьного интегрирования (из чего и состоит почти весь курс анализа в говновузах) не нужно, просто потому, что это никому не пригодится. Это настолько же бесполезно, как приёмы угадывания корней уравнений третьей степени.

Никто из математиков и почти никто из физиков и инженеров не столкнётся с необходимостью символьно считать неочевидные интегралы.
А те единицы, доли процента, что столкнуться пусть и изучают эту "науку". Остальным то зачем жрать говно и тратить на это столько ценного учебного времени? Ты не сможешь на это ответить, потому что ответ не лежит в рациональной плоскости: традиция.
>> No.93294 Reply
>>93292
Я вот изначально не собирался рассказывать очевидные вещи в десятый раз, но ты "развёл" меня на разговор.
Хитрый какой любитель картофана.
>> No.93295 Reply
>>93292
А ты ему просто приведи пример, где алгоритмы не работают и задача не автоматизирована до конца. Довала же интегралов, которые целиком и сразу не берёт вольфрам, и только разбив задачу на несколько отрывков и скормив их вольфраму, можно решить их сэкономив время (таки отличается от модели нажал-на-кнопку-робот-всё-рассчитал/не математика и т.д.)
>> No.93296 Reply
File: 3s.jpg
Jpg, 16.12 KB, 490×274 - Click the image to expand
edit Find source with google Find source with iqdb
3s.jpg
>>93293
> А вот специально изучать разные приёмы символьного интегрирования (из чего и состоит почти весь курс анализа в говновузах) не нужно, просто потому, что это никому не пригодится.
Да какие там приёмы, лол? Пособие в тридцать страниц текста - вот и все кошмарные адские жутчайшие приёмы. Древний как мир графоман Фихтенгольц смог написать про это всего лишь девяносто страниц. Зорич вообще в несколько страничек уложился.
Тем не менее тема нужная. Основные интегралы и физический смысл упоминаются даже у Бурбаки в "интегрировании мер".
> Никто из математиков и почти никто из физиков и инженеров не столкнётся с необходимостью символьно считать неочевидные интегралы.
Ну-ну. Загляни в трёхтомник Савельева и проверь это утверждение.
>> No.93297 Reply
>>93296
Если ненужной лабуде посвящено всего 30 страниц, это не значит, что их таки нужно изучать.
Ты придерживаешься точки зрения, что стакан соли надо съесть, потому что "так положено! как же без этого?!", а мне эта точка зрения, как и прочая архаика и мракобесие, чужда. Потому что подобная гадость портит людям мозги, вместо творчества и развития интеллекта, они занимаются рутиной семестрами и вырастают дефектными.
>> No.93299 Reply
>>93296
> трёхтомник Савельева
Это что ещё такое? Очередной учебник физики написанный всё в том же стиле "водофка & картофанчик"?
Если там читателей заставляют считать что-то вроде >>93231, то это, очевидно, аргумент против трёхтомника Савельева, а не в пользу задрачивания интегралов, лол.
>> No.93300 Reply
File: picture.jpg
Jpg, 11.55 KB, 340×255 - Click the image to expand
edit Find source with google Find source with iqdb
picture.jpg
>>93297
Ты вбил себе в голову, что интегралы не нужны. Уточняю: интегралы не нужны меньшинству, которое страдает "чистой математикой". Остальным они нужны. Не представляю, как обучить физика без дифуров и интегралов.
> Ты придерживаешься точки зрения, что стакан соли надо съесть, потому что "так положено! как же без этого?!"
Чушь.
> а мне эта точка зрения, как и прочая архаика и мракобесие
Мракобес здесь ты.
> посвящено всего 30 страниц
> они занимаются рутиной семестрами
Гм. Ну ок. Раз ты считаешь, что в университетах семестрами читают тридцать страниц, то объяснить тебе что-то уже нельзя. Оставайся наедине с придуманным тобой стаканом соли.
>> No.93303 Reply
>>93300
Я прихожу к выводу, что ты тупой, потому что не воспринимаешь письменный текст, хотя я пишу уж максимально подробно, у тебя "problems with comprehension".
> Ты вбил себе в голову, что интегралы не нужны.
Ну где я это написал, тупоголовое ты существо? Где?
Интегралы нужны, теория меры нужна, дифуры нужны и т.д.. Не нужно задрачивание символьного интегрирования. Не нужна рутина. От этого люди глупеют, как ты, например.
Символьное интегрирование, кроме очевидных интегралов, не нужно почти никому. Те единицы, кто с этим столкнётся на практике, пусть этому и учатся. А сейчас на гнилом мехмате, не говоря уже об инженерных шаражках, на это говно уходит целый семестр, который можно было бы потратить с пользой, тогда и невежд было бы поменьше.

Я утомился и на эту тему разговаривать больше не буду, уже достаточно сказано, чтобы зрители сделали выводы.
>> No.93304 Reply
>>93303
"Шаражки невежды говно мехмат рутина задрачивание ко-ко-ко".
Я с тобой разговариваю, а ты работаешь на зрителей и доказываешь, что лучше всех всё обо всём знаешь. Фу таким быть.
>> No.93306 Reply
>>93304
Я и не собирался тебя переубуждать, лол.
А вот неиспорченные детишки почитают нашу беседу и что-то да вынесут.
>> No.93307 Reply
>>93304
Не собирался не потому, что ты не няша, а потому, что человек, прошедший курс "высшей математики для инженеров", или старательно учившийся на мехмате уже потерян для нашей науки.
>> No.93308 Reply
>>93303
Символьное интегрирование есть везде же. Будь то MIT или Оксфорд, Гарвард или Кембридж, без разницы.
>> No.93310 Reply
>>93306
> Я и не собирался тебя переубуждать, лол.
Да ты и не можешь. Реальность не переспоришь - интегралы везде.
> уже потерян для нашей науки
> для нашей
Лол. То-то и оно, что для вашей.
>> No.93311 Reply
xxx: да вообще
xxx: когда учим математику - всю дорогу так, сначала нельзя того, потом можно, нельзя этого - и потом опять можно
xxx: неудивительно, что потом люди начинают употреблять наркотики и заниматься однополым сексом - они думают, что это тоже скоро станет можно - вот оно, пагубное влияние неправильно построенной программы образования
xxx: а так бы сразу с первого класса - тригонометрическая форма комплексного числа - фигак
xxx: ну, а к пятому классу и яблоки бы считать научились

c бора
>> No.93329 Reply
Сап, няши.
Всем добра.
А картофан-куну двойную порцию.
Такие дела.
>> No.93331 Reply
>>93307
Няша, собираюсь поступать на физмат Карлова Университета. Ничего не знаешь - если закончу, буду потерян для математики или нет?
>> No.93333 Reply
File: i-robot.png
Png, 243.92 KB, 750×321 - Click the image to expand
edit Find source with google Find source with iqdb
i-robot.png
>>93329
И тебе добра, неймфаг!
>> No.93335 Reply
File: 1362327346130.jpg
Jpg, 45.68 KB, 375×500 - Click the image to expand
edit Find source with google Find source with iqdb
1362327346130.jpg
Раз уж упомянули инженеров, то вот вам прохладная.
Линейной алгебры нет, оптимизации нет, численных методов нет, матфизики нет. Дифуров нет, кроме тупо механического разделения переменных, полных дифференциалов, постоянных коэффициентов, ну вы поняли.
Нужно ли говорить, что про общую топологию никто не заикался, зато дрочке неопределенных интегралов таки да, таки семестр посвящен был. Как так вообще, скажите мне?
>> No.93336 Reply
>>93335
Кофе - мужского рода. А среднего - говно и министерство образования.
Хуле я как препод могу сделать, если в стандарте написано - ебать мозги интегралами неопределенными.
Олсо, мабуть эти темы вынесены в другие курсы?
Если пилить мимо стандарта, придет ональная комиссия и пиздец.
>> No.93338 Reply
File: normal_1311383297_bigi4c2b0be0ee99b.jpg
Jpg, 52.31 KB, 466×600 - Click the image to expand
edit Find source with google Find source with iqdb
normal_1311383297_bigi4c2b0be0ee99b.jpg
>>93336
Можно ебать мозги неопределёнными интегралами как набором белиберды, если учишь не математиков. А можно рассказать вот про это ruwiki://Когомологии_де_Рама и ебать мозги с умом.
>> No.93339 Reply
>>93336
> Олсо, мабуть эти темы вынесены в другие курсы?
Неа, все скидано кучей в курс "высшая математика", за его пределами только таблицы-коэффициенты-формулки, пояснения за суть протекающих процессов на пальцах.
Хотя, нам декан пытался в рамках курса "теоретические основы конечноэлементного моделирования" рассказать, зачем нужны матрицы и дать хоть какое-то представление о теории упругости, но это, честно сказать, и смешно и грустно. Не летайте на пуперджетах.
>> No.93340 Reply
>>93339
На каком ты сейчас курсе?
>> No.93341 Reply
>>93340
Я уже давно закончил(знаю что лох) и переучиваюсь на прикладноблядь теперь.
>> No.93342 Reply
>>93338
Хеллоу, у людей линейной алгебры нет, вы чего вообще?
>> No.93343 Reply
File: 1.jpg
Jpg, 39.64 KB, 737×402 - Click the image to expand
edit Find source with google Find source with iqdb
1.jpg
>>93342
А как ты им про якобианы и вронскианы рассказываешь? Как вводишь операторы Гамильтона и Лапласа?
>> No.93344 Reply
File: 3tksnkqj3rei.jpg
Jpg, 41.12 KB, 479×792 - Click the image to expand
edit Find source with google Find source with iqdb
3tksnkqj3rei.jpg
>>93343
Я не препод-кун. Инженерам дальше трехмерных пространств заглядывать неположено, поэтому все выписывается в виде окончательных формулок.
>> No.93348 Reply
>>93338
> неопределёнными интегралами
> ruwiki://Когомологии_де_Рама
You do it wrong.
>> No.93349 Reply
>>93348
R u sure?
>> No.93350 Reply
Доброкафедра, привет. У меня вопрос по определениям.
Пусть есть множества A и B, которые являются линейными пространствами. Если их декартово произведение AxB - линейное пространство, то AxB называется тензорным произведением A и B. Так?
>> No.93352 Reply
>>93350
> множества A и B, которые являются линейными пространствами.
Множества не являются векторными пространствами.
> Так?
Нет, λx⊗y=x⊗λy, например.
Для тензорное произведение определяется своим универсальным свойством:
Векторное пространство U вместе с билинейным отображением f: AxB→U называется тензорным произведением векторных пространств A и B, если для любого билинейного отображения g:AxB→W существует единственное линейное отображение r:U→W, что g=r∘f.
>> No.93358 Reply
>>93336
> Кофе - мужского рода.
Этот род достался ему от форм "кофий" и "кофей". А метро тоже был он, так как метрополитен.
> Если пилить мимо стандарта, придет ональная комиссия и пиздец.
Хм, у нас вроде сильно пилят мимо стандарта и ничё.
>> No.93359 Reply
Аноны, поясните, есть такое понятие как прикладная математика? Что оно обозначает?
>> No.93364 Reply
>>93358
Насколько сильно?
>> No.93375 Reply
>>93364
Курс был в виде сборной солянки несвязанных тем да ещё дублирующихся с другими курсами. Поэтому темы повыкидывали и оставили одну.
>> No.93381 Reply
>>93331
Я немного преувеличил, никто не потерян для математики, разумеется.
(но некоторым придётся долго переучиваться, если они захотят ей заниматься, лол)
А про этот университет ничего сказать не могу. А почему ты решил именно туда? В магистратуру/пхд/бакалавриат?

>>93350
Нет, это совершенно разные вещи.

>>93359
> есть такое понятие как прикладная математика?
Нет, такой науки нет. Традиционно "прикладными" называют разделы математики, где нет никакой глубокой теории, но много вычислений.
Умные люди говорят, что не бывает прикладных наук, а бывают приложения наук. Тот, кто не знает самой науки, не сможет найти и приложений, лол.
>> No.93383 Reply
>>93381
> Нет, такой науки нет.
А специальность такая есть?
>> No.93385 Reply
Мне кажется или ты подменяешь топологией всю математику?
>> No.93386 Reply
>>93335
Будь добр, скинь перечень тем и название учебника, по которому занимаетесь.
>> No.93389 Reply
>>93381
Ну, я вообще собираюсь в Чехии учиться - тут люди няшнее, по голове стучат меньше, и в американский университет отсюда съебать на магистратуру проще. Карлов просто хороший европейский, на одном уровне с Сорбонной, к примеру. Можешь посоветовать, как становиться математиком? Или просто советов мудрых?
>> No.93390 Reply
>>93386
Не могу скопипастить, гуглится. Стыдно показывать кафедре, насколько херовое у меня образование.
Учебник - даже не Фихтенгольц+Демидович. В наше время, помнится теоретическим учебником был Бермант, а задачник - Берман, я это пару запомнил. А сейчас там все окончательно ебнулись и перешли на книжки собственного издания. Такие дела.
>> No.93410 Reply
>>93390
> помнится теоретическим учебником был Бермант
Никогда не понимал, зачем нужно грузить инженеров псевдоматематическими доказательствами. Кому это надо? Лучше дать список готовых формул и приложить к нему список литературы, в которой эти формулы выводятся, чтобы тот, кому интересно, мог прочитать.
>> No.93417 Reply
>>93335
Знакомая ситуация, бро. Только в конце четвертого курса узнал зачем на практике могут быть нужны матрицы, СЛАУ и т.п. Прошлым летом читал книгу по векторному исчислению, наконец узнал что такое rot, div, оператор Лапласа и Гамильтона и т.п. Занимательно было до этого на втором-третьем курсе читать книги по гидродинамике и теплообмену, половину там не понимая. Недавно познакомился с методами решения нелинейных уравнений - очень полезная штука, а я только сейчас с ней разобрался. Вот так у нас инженеров учат. Я даже и не знаю, хорошо это или плохо. Большинству ведь это особо не надо. С другой стороны, если вдруг встанет задача нестандартная - нихрена не сможешь.
>> No.93422 Reply
>>93417
> наконец узнал что такое rot, div, оператор Лапласа
У нас эт на первом курсе было, векторн. анализ инженерам обязон же.
>> No.93423 Reply
>>93390
> помнится теоретическим учебником был Бермант
Для первого курса - вполне норм. Первый курс? Всё прошли?
>> No.93425 Reply
>>93423
Он два года был размазан (он плюс элементарный тервер и векторный анализ). По моим воспоминаниям, чисто на него ушло полтора, благорадя семестру неопределенных интегралов
В общем, друзья, давайте закрывать эту тему. Я, честно сказать, запостил свою историю только чтобы спросить у человека с аватаркой - действительно ли он считает, что неопределенные интегралы настолько незаменимы в подготовке инженеров, что ради них вот это все.
>> No.93426 Reply
File: 2104020179.jpg
Jpg, 30.84 KB, 300×429 - Click the image to expand
edit Find source with google Find source with iqdb
2104020179.jpg
>>93425
Неопределённые интегралы нужны, и поливать их грязью - глупость. Однако изучать их целый семестр не нужно. Вероятно, так много времени было выделено потому, что деканат желал, чтобы все студенты умели пользоваться неопределёнными интегралами. Если так, то это грубый просчёт. Студенты, тупые настолько, что на усвоение тридцати страниц текста им потребен целый семестр, не смогут стать инженерами, поэтому о них можно не заботиться. А умные студенты сойдут с ума, растягивая элементарщину на сорок занятий.
>> No.93429 Reply
File: qs114y.jpg
Jpg, 30.25 KB, 501×600 - Click the image to expand
edit Find source with google Find source with iqdb
qs114y.jpg
>>93426
> не смогут стать инженерами
Хорошо, что ты такого высокого мнения об инженерах. Я уверен, что ты не про рашку. Потому что в рашке профильную олимпиаду быдловузик чаще всего тащит.
Большинство выпускников, конечно, работает не по специальности. Но кто-то проектирует суперджет. Такие дела.
> Однако изучать их целый семестр не нужно.
Ок, так и постановим.
>> No.93433 Reply
>>93426
Неопределенные интегралы не нужны.
>> No.93437 Reply
>>93433
Больше неопределённых интнгралов богу неопределённости!
>> No.93443 Reply
>>93385
> подменяешь топологией всю математику?
Видимо, этот вопрос был адресован мне.
Нет, топология это примерно треть математики, не вся.

>>93389
Когда приедешь на место и начнётся учебный процесс, постарайся посещать научные/студенческие семинары на интересные тебе темы, чем больше тем лучше. Конечно, поначалу ты ничего не будешь там понимать, но это не страшно: запоминай ключевые названия и идеи, а потом старайся разобраться, что это, по англвики/книжкам. И приставай с вопросами ко всем.

Алсо, не засиживайся в общаге. В математике учебники на втором плане, основную порцию знаний студент приобретает в беседах, на докладах, слушая чью-то беседы и т.д.

>>93417
> наконец узнал что такое rot, div
Тебя ждёт ещё много приятных сюрпризов, когда ты почитаешь книжку про анализ на многообразиях и дифференциальные формы. Потому что роторы и дивергенции это язык даже не двадцатого, а девятнадцатого века.
>> No.93445 Reply
>>93410
> Лучше дать список готовых формул
Никогда не понимал, откуда берутся люди, утверждающие, что в математике есть формулы? В математике принципы, идеи, теоремы, а так же методы решения задач из разных классов задач. Формулы только в химии и учебнике физики 7-го класса для слаборазвитых.
>>93417
Это просто пушка. А я в подозрениях из-за местных анонов%% и неймфагов%%, нужны ли мне цермело-френкели и когомологии де Рама.
>>93422
У нас - на третьем(собственно я на третьем на данный момент.). В курсе т.н. функционального анализа. На первом втором курсе был по идее типа классический матан и конкретно дифференциальных операторов всяких таких не было.
>> No.93459 Reply
>>93443
> Тебя ждёт ещё много приятных сюрпризов, когда ты почитаешь книжку про анализ на многообразиях и дифференциальные формы. Потому что роторы и дивергенции это язык даже не двадцатого, а девятнадцатого века.
Я начал читать, но по мне это слишком абстрактно. Не осилил короче. С тензорным анализом та же байда. Слишком много всяких индексов, размерностей, черт ногу сломит. Короче, как раз анекдот такой есть:
> Инженер три часа просидел на лекции математика про многомерные пространства. В конце он, очень огорченный, подошел к лектору и сказал:
> - Извините, я хотел бы хоть немножко представить себе предмет вашей лекции. Но я не могу вообразить сферу в девятимерном пространстве!
> - Это же очень просто, - ответил ему математик, - вообразите сферу в N-мерном пространстве, а затем положите N равным девяти.
>> No.93464 Reply
>>93459
Зачем ты пытаешься визуально представить абстракции? Ты же не представляешь себе числа, когда считаешь, так зачем представлять себе векторы, матрицы и тензоры? Это почти такие же "числа", так что и представлять их нет смысла. И отображения не нужно представлять. И операторы. Вообще, с любым абстрактным математическим объектом лучше всего работать как со словом. Если что-то называется сферой, то это не значит, что оно визуально выглядит как мячик. Бери пример с программистов. Они не представляют себе форму классов, форму объектов и форму лямбда-функций, они просто оперируют всем этим.
> Слишком много всяких индексов, размерностей, черт ногу сломит.
Это дело привычки. Читай больше, и скоро будешь хорошо воспринимать эти адские закорючки.
> С тензорным анализом та же байда
Работай над определениями. Думай о них, пытайся переформулировать их так, чтобы тебе было проще.
>>93352 - вот определение тензорного произведения. Билинейное отображение - это отображение Ф такое, что
Ф(x+x',y) = Ф(x,y) + Ф(x',y)
Ф(x,y+y') = Ф(x,y) + Ф(x,y')
Ф(ax,y) = aФ(x,y)
Ф(x,yb) = Ф(x,y)b
Тензор - это элемент тензорного произведения. Тензор ранга (n,0) можно считать обычной полилинейной функцией (линейной по каждому своему аргументу; линейная и билинейная функции - это полилинейные функции).
>> No.93465 Reply
>>93459
> тензорным анализом
Это так анализ на многообразиях столетней давности называется, лол.
>> No.93466 Reply
>>93445
Лолчто? В математике нет формул? А что такое по-твоему формула, пушкарь?
>> No.93468 Reply
Есть тут те, кто считает в воображение?
>> No.93474 Reply
>>93468
Что-что делает?
>> No.93476 Reply
>>93464
Тэкс. Тензоров у нас не было. А билинейные формы были, очень даже. Может у нас были тензоры, но я не знал, что это они? Само понятие тензора не устарело? Так как с билинейной формой соотносится тензор и тензорное произведение?
>> No.93477 Reply
>>93466
Ну есть, положим, но они от всей математики мало занимают.
>> No.93478 Reply
>>93468
Я недавно считал в ее воображение.
>> No.93480 Reply
File: Heat_eqn.gif
Gif, 116.08 KB, 200×136 - Click the image to expand
edit Find source with google Find source with iqdb
Heat_eqn.gif
Такой вопрос. Есть функция числовая U(x,t) либо U(x,y,t). Здесь x - точка на прямой, (x,y) - точка на плоскости, а t - время прошедшее с точки отсчёта, взятой как t=0. Хочется что? Хочется наблюдать двумерный график U(x,t) от x, который бы анимацией менялся, изменяя t от нуля до какого-то момента t=a. Также хочется трёхмерный график U(x,y,t) от (x,y), который бы также менялся анимацией, изменяя t от нуля до какого-то момента t=a. При этом было бы замечательно, если бы U могла быть подана на вход в виде функционального ряда или интегралов от всяких там ядер Пуассона. Было бы вообще круто, если б можно было U(x,y,z,t) тоже как-то визуализовать. На чём это возможно? На пикрелейтеде пример ожидаемого результата.
>> No.93482 Reply
Аноны, есть проблемка. Учусь я в 7 классе, увлекаюсь математикой, решаю различные олимпиадные задачки дома и на олимпиадах. В школьных олимпиадах можно задачки классифицировать: есть связанные с логикой (на истинность и ложность высказываний), есть геометрические задачки, есть задачки, которые решаются рисованием графов, есть комбинаторные и т.д. Такие задачи я более менее иногда решаю, но есть определенный круг задач, в который я не могу совсем, например: 1) Докажите, что сумма 1^3+1^3+...+99^3 делится на 100. 2) Делится ли число 111...1 (81 раз этих единичек) на 81. 3) Докажите, что n^2+1 не делится на 3 ни при каком натуральном n. 4) Докажите, что остаток от деления простого числа на 30 есть простое число или единица. Ну и так далее. Вообщем что с этими "делится/не делится, остаток/не остаток" делать я не знаю, но научиться подобного рода вещи решать я должен, но вот как? Может почитать чего, посмотреть? Я понимаю, что вы какие-то сложные и непонятные слова элементарщиной зовет, а тут такое, но все-таки нуб искренне надеется на ваши годные советы, просто обратиться за помощью мне больше не к кому.
>> No.93483 Reply
>>93480
OpenGL.
>> No.93485 Reply
>>93482
Вычисления - не математика. Вычисления - удел роботов, в том числе и биороботов. Математик, вообще говоря, - не биоробот. Математик доказывает, доказывает интересные ему вещи. Чтобы научиться доказывать, нужно прокачать логику (логику высказываний и логику предикатов). Перед началом доказательства нужно записать определения сущностей, с которыми работаешь, и записать то, что хочешь доказать. И лишь затем можно приступать к поиску доказательства.
Зорич. "Математический анализ 1" — http://yadi.sk/d/ES1nSogn0W-f5 - здесь краткое введение в логику
Шенфилд. "Математическая логика" — http://yadi.sk/d/vKZ3PM9u0W4Td
Гильберт, Аккерман. "Основы теоретической логики" — http://yadi.sk/d/rXvA2HTK0W1F6
Виноградов. "Теория чисел" — http://topology.math.csu.ru/library/posob/terch/VINOGRADOV_NUMBER_THEORY.PDF
>> No.93486 Reply
>>93482
это изучает теория чисел.
>>93485
тут ссылку дали
>> No.93488 Reply
>>93474
Производит вычисления в воображение.
>> No.93490 Reply
>>93488
Может быть, в воображении?
>> No.93495 Reply
>>93485
Не нужно. Матлогику не математики учат.
>> No.93496 Reply
>>93495
Умение в матлогику - необходимый признак математика.
>> No.93497 Reply
>>93496
Необходимый признак матлогика (это философы).
Математики умеют в матлогику по наитию. Учить не обязательно. Зато есть куча других вещей, которые математики должны знать.
>> No.93498 Reply
>>93497
> Математики умеют в матлогику по наитию. Учить не обязательно.
Знаешь, в чём разница между первобытным шаманом и нейрохирургом? Впрочем, о чём тут говорить.
Иди к лешему.
>> No.93499 Reply
>>93498
Иди у математиков спроси про какую-нибудь матлогическую фигню - они тебя пошлют. Опыт-таки весомее всяких рассуждений. :3
>> No.93500 Reply
File: Ты-упорот-шоле.jpg
Jpg, 64.30 KB, 684×596 - Click the image to expand
edit Find source with google Find source with iqdb
Ты-упорот-шоле.jpg
>>93499
Это к каким математикам предлагаешь сходить?
>> No.93501 Reply
>>93500
Ну не знаю. Достойные математики не знают, что такое эрбрановская функция инфа 99%. У Девочки волшебницы или Бородатого фрика поспрашивай насчет них. Они получше разбираются. Если чо, я дилетант.
Просто у меня знакомый с мехмата угорает по матлогике, считает себя знающим математику и топологию, сдал(!) мехматиский курс алгебраической топологии и не знает, что такое клеточное пространство, но знаком с моделями Крипке. Вывод: частое занятие формальной чепухой портит людей.
>> No.93503 Reply
>>93501
Сперва просмотри книжки, которые я советую, а потом лезь со своим "не нужно".
Я и сам не знаю, что такое эрбрановская функция, лол
>> No.93504 Reply
>>93503
Шенфиилд большеват. Гильберт-Аккерман - старьё.
Мне известна Манинская книжка по матлогике и Расёва-Сикроский (углубленно, но хороший математический подход, советуют). Из классических читал Мендельсона. Он небольшой.
>> No.93505 Reply
>>93504
Очепяток много, да.
> Шенфилд
> Сикорский
>> No.93507 Reply
>>93445
> У нас - на третьем(собственно я на третьем на данный момент.). В курсе т.н. функционального анализа. На первом втором курсе был по идее типа классический матан и конкретно дифференциальных операторов всяких таких не было.
Спасибо, ты меня удивил. Я, если честно, слегка шокирован.
>> No.93510 Reply
>>93483
Уффф, а готового мат-ПО нету?
>> No.93515 Reply
>>93490
Какая мне разница? Ты же понял вопрос, в чём проблема? Мне просто интересно узнать есть те, кто "рисует" вычисления в воображении и всё, что сложного то? Нет, найдётся самый умный, который сначала расскажет, что во мне не так, а потом расскажет почему я спрашиваю ерунду и только потом напишет ответ на вопрос.
>> No.93522 Reply
File: 2342.png
Png, 26.84 KB, 602×485 - Click the image to expand
edit Find source with google Find source with iqdb
2342.png
Анон, помоги мне решить задания.
>> No.93523 Reply
File: picture.jpg
Jpg, 11.55 KB, 340×255 - Click the image to expand
edit Find source with google Find source with iqdb
picture.jpg
>>93522
Анон >>93232 уже помог тебе, нужно только прочитать эти несчастные тридцать страниц и воспользоваться алгоритмами. Ты хочешь, чтобы решили задания за тебя? Но ведь это твои задания и твоя учёба, зачем нам делать за тебя твою работу?
>> No.93524 Reply
Снова пропиарю книжку Львовского, вдруг кто-то ещё не скачал: http://rghost.ru/43906548
>> No.93525 Reply
File: smoke.jpg
Jpg, 26.10 KB, 400×544 - Click the image to expand
edit Find source with google Find source with iqdb
smoke.jpg
>>93352
> Множества не являются векторными пространствами.
Но векторные пространства при стандартном множествами являются.
Есть теория векторных пространств, её модели строятся в расширении теории множеств в виде кортежей, где первый член носитель, а второй член сигнатура.
>>93498
> Знаешь, в чём разница между первобытным шаманом и нейрохирургом?
У математтиков и есть шаманская научная логика. А в честную игру логический вывод они не могут.
>> No.93526 Reply
>>93523
Я хочу чтобы мне объяснили как сделать из этих интегралов табличные. Я уже сделал 15 заданий, а остальные 9 мне не под силу.
>> No.93527 Reply
По поводу мат. логики.
Это отдельная наука со своими определениями, своими теоремами своими мотивировками и т.д. К умению последовательно рассуждать она относится ни больше и ни меньше, чем другие разделы математики.
Чтобы заниматься алгеброй/геометрией/анализом и т.д. никакой мат.логики знать не нужно и наоборот.
Большинство математиков логику знают на самом элементарном уровне, а большинство логиков, соответственно, не знают никакой математики.

Тем не менее, пересечение математики и мат.логики не совсем пусто, теория моделей имеет некоторые содержательные приложения к алгебраической геометрии, кому интересно гуглите "model theory and algebraic geometry"
>> No.93536 Reply
>>93522
5. Числитель - это производная знаменателя. Интеграл равен ln|2sin(x) - 3 cos(x)| + const.
8. t = 1+5tg(5x); dt = 25dx/cos^5x; интеграл от (1/25)t^(1/3)dt
9. Расписываешь на разность интегралов. Первый даст логарифм, во втором знаменатель - производная арктангенса, берёшь арктангенс за новую переменную.
12. По частям же, два раза. t = x+1.
Ну и так далее. Смотри у Письменного, ссылка выше.
>> No.93542 Reply
File: 1319657792185.jpg
Jpg, 140.14 KB, 1037×691 - Click the image to expand
edit Find source with google Find source with iqdb
1319657792185.jpg
>>93536
Спасибо, анон. Ты мне очень помог.
>> No.93556 Reply
Единственный способ научиться - это решать больше.
Скачай архив журнала Квант, там куча олимпиадных задач разного уровня с решениями.
Также в разных номерах бывают подборки задач по конкретным темам.

Есть видеолекции http://rutracker.org/forum/viewtopic.php?t=3972234 и по ссылкам.
Дальше прорешивай задачи с турнира городов, городских, областных, всероссийских олимпиад.

http://rutracker.org/forum/viewtopic.php?t=4164275
http://rutracker.org/forum/viewtopic.php?t=1511047
http://problems.ru/
>> No.93557 Reply
>>93482
>>93556
Вот этому анону
>> No.93564 Reply
>>93556
> Единственный способ научиться
Чему?
>> No.93567 Reply
Так всё-таки почему 0.(9)=1?
>> No.93568 Reply
>>93556
Спасибо, вероятно это то, что мне нужно.
>> No.93569 Reply
>>90593
Расскажите, а какие последние открытия в математической области? Что происходит в математическом мире? Какие области развиваются? Очень интересно. Кстати, реально ли заниматься математикой в каких-либо интересных областях, если твои познания в целом не так то велики, но чувствуешь, что определенные способности есть.
Студентота 1 курс
>> No.93570 Reply
>>93567
Читай посты треда, этого и предыдущего.
>> No.93571 Reply
>>93567
Алсо, доказательства на вики.
>> No.93573 Reply
>>93567
По кочану.
>> No.93574 Reply
>>93573
Не понимаю.
>> No.93575 Reply
>>93567
Потому что 0.(9) = 1 - (0.1)^n, n -> infinity
>> No.93576 Reply
>>93575
Что за infinity?
> 0.(9) = 1 - (0.1)^n
если u = a - b, то a>u. 0.(9) != 1
>> No.93577 Reply
>>93576
Узнай уже наконец определение предела и все встанет на свои места.
>> No.93578 Reply
>>93574
Это оффициальный ответ кафедры. Определение термина "кочан" дано в предыдущих тредах.
>> No.93579 Reply
>>93577
Какого из опредлений предела?
>> No.93580 Reply
File: 1362666224856.png
Png, 1.07 KB, 300×20 - Click the image to expand
edit Find source with google Find source with iqdb
1362666224856.png
>>93575
Вопиюще неправильная строка текста - как с формальной, так и с логической точки зрения. Ужас же, больше не доказывай ничего.
>> No.93581 Reply
>>93580
И что там неправильно?
>>93579
Любого.
>> No.93582 Reply
0.(9) и 1.(0) - это бесконечные десятичные дроби. Есть отображение из множества дробей на точки на прямой. Это отображение не взаимно-однозначное. Сюръективное, но не инъективное. Две дроби равны, если им соответствует одна и та же точка. 0.(9) и 1.(0) отображаются в одну и ту же точку, потому и равны.
>> No.93584 Reply
>> No.93585 Reply
>>93581
> И что там неправильно?
Для начала то, что переход к пределу уже совершён.
>> No.93586 Reply
>>93585
Но в стандартных курсах матана первого курса выводится сначала арифметика бесконечно малых, а на их основе арифметика пределов, поэтому я подумал, что так будет понятнее.
>> No.93588 Reply
>>93586
Не бывает "стандартных курсов" матана. Бывают либо курс анализа, либо мракобесие.
>> No.93589 Reply
File: 13615651529101.jpg
Jpg, 95.51 KB, 800×640 - Click the image to expand
edit Find source with google Find source with iqdb
13615651529101.jpg
>>93588
Ух кокойты категоричный. Жи есть, нет бога кроме Аллаха!
>> No.93592 Reply
>>93585
Строка верная. То есть она формально написана безошибочно и верна. Но она не является доказательством.
>> No.93594 Reply
>>93589
Именно так. Третьего нет.
>> No.94303 Reply
Доброчан, математики!
Пожалуйста, у кого есть опыт изучения дифференциальных уравнений, поделитесь, к какому учебнику лучше всего обратиться.
Нужно что-то простое, довольно подробное и последовательное - чтобы "понятно было". Желательно, чтобы учебник был попыткой автора написать хорошую книжку, а не просто книжку, которые иногда делаются для отчетности.
Зубрить ничего не собираюсь, да и не умею. Нужно навести в голове порядок и комфортно себя чувствовать, как можно реже прибегая к двоемыслию.
Надеюсь, что-то близкое к описанному мной идеалу все-таки существует.
//к: улыбкой надеялся - воистину!
>> No.94306 Reply
Нашел! Не книжку нашел, а нашел, где ее, может быть, можно найти.
Вот тут список
http://pay.diary.ru/~eek/p48302307.htm#
>> No.94866 Reply
Привет, анон.
Так случилось, что пока я болел в моя группа изучала интегралы. И вот теперь, когда моя руппа начала учить диф. уравнения, а я в свою очередь все-таки выздоровел, я понял, что в интегралах я полное дно. Посоветуй какой-нибудь литературы, анон, прошу тебя.
Алсо, желание и кое-какие способности к изучению есть.
>> No.95319 Reply
>>93293
> почти никто из физиков и инженеров не столкнётся с необходимостью символьно считать неочевидные интегралы
НУ ЭТО ВООБЩЕ ПУШКАнейтронная
>> No.95673 Reply
Следующий тред есть?
>> No.95712 Reply
>>90593

Ребята, может кто-нибудь может объяснить, как это работает, и почему?
http://cs521306.vk.me/u16000386/doc/5922a5b05127/file.gif

// математико-нуб
>> No.96601 Reply


Password:

[ /tv/ /rf/ /vg/ /a/ /b/ /u/ /bo/ /fur/ /to/ /dt/ /cp/ /oe/ /bg/ /ve/ /r/ /mad/ /d/ /mu/ /cr/ /di/ /sw/ /hr/ /wh/ /lor/ /s/ /hau/ /slow/ /gf/ /vn/ /w/ /ma/ /azu/ /wn/ ] [ Main | Settings | Bookmarks | Music Player ]